1 Introduction

This work continues our program on the theory of transportation of closed differential forms. The current manuscript studies actions defined on paths of closed differential forms, introduces various distances and improves on the study in [9] (for related work more centered on the symplectic case where \(k=2,\) see [10]). We denote by \(\Lambda ^{k}\) or \(\Lambda ^{k}(\mathbb {R}^{n})\), the set of exterior k-forms over \(\mathbb {R}^{n}\) (k-covectors of \(\mathbb {R}^{n}\)).

Consider a convex (in fact contractible will be sufficient) open bounded set \(\Omega \subset \mathbb {R}^{n}\) and denote by \(\nu \) the unit outward vector to the boundary \(\partial \Omega \). Let d denote the exterior derivative operator on the set of differential forms on \(\Omega \) and let \(\delta \) denote the adjoint (or co-differential) of d. Let \(\bar{f}_{0},\bar{f}_{1}\) be two-closed k-forms on \(\Omega \) (i.e. their distributional differential \(d\bar{f}_{0}\) and \(d\bar{f}_{1}\) are null) and the compatibility condition

$$\begin{aligned} (\bar{f}_{1}-\bar{f}_{0})\wedge \nu =0\quad \hbox {on}\quad \partial \Omega \end{aligned}$$
(1.1)

is satisfied when \(1\le k\le n-1\) while we impose that

$$\begin{aligned} \int _{\Omega }(\bar{f}_{1}-\bar{f}_{0})dx=0 \end{aligned}$$
(1.2)

when \(k=n.\) Accordingly, we denote by \(\mathcal {H}\), the set of k-forms \(h\in L^{1}(\Omega ;\Lambda ^{k})\), which are closed in the weak sense, and such that when \(1\le k\le n-1\) then

$$\begin{aligned} (h-\bar{f}_{0})\wedge \nu =0\quad \hbox {on}\quad \partial \Omega \end{aligned}$$

while when \(k=n\) it is rather required that

$$\begin{aligned} \int _{\Omega }(h-\bar{f}_{0})dx=0. \end{aligned}$$

This is a subspace of the separable Banach \(L^{1}(\Omega ;\Lambda ^{k})\). If \(s\rightarrow f_{s}\) is a path in \(\mathcal {H}\), since on contractible domain every closed form is exact and \(s\rightarrow -\partial _{s}f_{s}\) remains a path of closed forms, there exists a path \(s\rightarrow A_{s}\) of \((k-1)\)-forms such that \(-\partial _{s}f=dA.\) Let \(p\in (1,\infty )\). In fact, we are interested in pairs (fA) such that

$$\begin{aligned} A\in L^{p}\left( (0,1)\times \Omega ;\Lambda ^{k-1}\right) ,\quad f\in L^{p}\left( (0,1)\times \Omega ;\Lambda ^{k}\right) ,\quad (f_{0},f_{1})=(\bar{f}_{0},\bar{f}_{1}) \end{aligned}$$
(1.3)

and

$$\begin{aligned} \partial _{s}f+dA=0\;\;\hbox {in}\;\;(0,1)\times \Omega \qquad \hbox {and}\qquad A\wedge \nu =0\;\;\hbox {on}\;\;[0,1]\times \partial \Omega \end{aligned}$$
(1.4)

in the weak sense (cf. Definition 2.2). The variable s has, a priori, no physical meaning and only serves as an interpolation variable between two prescribed closed forms. Let us denote by \(P^{p}(\bar{f}_{0},\bar{f}_{1})\) the set of pairs (fA) such that (1.3) and (1.4) holds.

Let \(c: \Lambda ^{k} \times \Lambda ^{k-1} \rightarrow [0,\infty ]\) be a lower semicontinuous function such that when \(\omega \in \Lambda ^{k}\), \(\xi \in \Lambda ^{k-1}\) and \(c(\omega , \xi )<\infty \) then

$$\begin{aligned} c(\omega , \xi )=0 \;\;\; \hbox {if and only if} \;\;\; \xi =0. \end{aligned}$$
(1.5)

In order for c to induce a Riemannian or Finsler type metric, we further assume that

$$\begin{aligned} c(\omega , \lambda \xi )=|\lambda |^{p} c(\omega ,\xi ). \end{aligned}$$
(1.6)

For \(f \in L^{1}(\Omega ; \Lambda ^{k})\) and \(A \in L^{1}(\Omega ; \Lambda ^{k-1})\) we set

$$\begin{aligned} ||A||_{f}= \left( \int _{\Omega }c\left( f, A \right) dx \right) ^{\frac{1 }{p}} \end{aligned}$$
(1.7)

and define Finsler type metrics

$$\begin{aligned} M_{p}(\bar{f}_{0}, \bar{f}_{1}):= \inf _{(f, A)} \left\{ \int _{0}^{1} ||A_{s}||_{f_{s}} ds \; \Big | \; (f, A) \in P^{p}(\bar{f}_{0}, \bar{f}_{1})\right\} . \end{aligned}$$
(1.8)

By Jensen’s inequality

$$\begin{aligned} \left( \int _{0}^{1} ||A_{s}||_{f_{s}} ds \right) ^{p} \le \int _{0}^{1} ||A_{s}||_{f_{s}} ^{p} ds. \end{aligned}$$

But using the standard “reparametrization of constant length” (cf. Lemma 5.2), one shows that in fact

$$\begin{aligned} M_{p}^{p}(\bar{f}_{0}, \bar{f}_{1})= \inf _{(f, A)} \left\{ \int _{0}^{1} ||A_{s}||_{f_{s}}^{p} ds \; \Big | \; (f, A) \in P^{p}(\bar{f}_{0}, \bar{f}_{1})\right\} . \end{aligned}$$
(1.9)

When \(c(f,A)=|A|^{p}\), \(p\in [1,\infty )\) and \(rp=r+p\) then a sufficient condition for (fA) to minimize (1.8) is (cf. [9])

$$\begin{aligned} f_{s}=(1-s)\bar{f}_{0}+s\bar{f}_{1},\;\;\bar{f}_{1} -\bar{f}_{0}+dA=0,\;\;A_{s} \equiv \delta g|\delta g|^{r-2},\;g\in W^{1,r}(\Omega ;\Lambda ^{k}),\;dg\equiv 0 \end{aligned}$$
(1.10)

and so in this case, A is time independent. Further restricting p to \((1,\infty )\) turns (1.10) into a necessary condition, which uniquely characterizes the minimizers.

By Sect. B.3, any convex function \(c:\Lambda ^{k} \times \Lambda ^{k-1}\rightarrow [0,\infty )\) (hence assuming only finite values) satisfying (1.5) and (1.6) must be independent of \(\omega .\) This is precisely the case already studied in [9]. This motivates our desire to study cost functions which take on infinite values. What matters the most in the choice of our cost function is the scaling condition (1.6), which is necessary to induce a metric.

An example of \(c(\omega ,\xi )=G(|\omega |,\xi )\) taking infinite value and studied in Sect. B.1 is

$$\begin{aligned} c(\omega ,\xi )=\left\{ \begin{array}{ll} {\frac{|\xi |^{p}}{p\;(1-|\omega |^{2})^{\frac{p-1}{2}}}} &{} \quad \text {if } \;|\omega |<1\\ 0 &{} \quad \text {if}\;\;\xi =0\;\;\;\hbox {and}\;\;\;|\omega |=1\\ \infty &{} \quad \text {if}\;\;(\xi \not =0\;\;\;\hbox {and}\;\;\;|\omega |=1)\;\;\;\hbox {or}\;\;(|\omega |>1). \end{array} \right. \end{aligned}$$
(1.11)

We can also consider cost functions of the form \(G(|\omega |,\xi )+H(\xi )\),obtained by adding to the c in (1.11) a smooth function H. One could replace the denominator in the cost in (1.11) by \(p\;(M-|\omega |^{2})^{\frac{p-1}{2}}\), where M is a positive parameter. In this case, any minimizing path (fA) in (1.9) must satisfy the requirement \(|f| \le M.\)

Let us for a moment keep our focus on the case \(k=2.\) Given a non-degenerate closed smooth 2-form f, there exists a 1-form w such that

$$\begin{aligned} A=w\,\lrcorner \,f\quad \hbox {and so}\quad dA=\mathcal {L}_{w}f, \end{aligned}$$
(1.12)

where \(\mathcal {L}_{w}\) is the Lie derivative acting on the set of 1-form (w has been identified with a vector field). A variant of (1.9) is

$$\begin{aligned} \inf _{(f,w)}\left\{ \int _{0}^{1}||w\,\lrcorner \,f||_{f_{s}}^{p} ds\;\;\Big |\;\;\partial _{s}f+\mathcal {L}_{w}f=0\right\} , \end{aligned}$$
(1.13)

where the infimum is performed over the set of (fw) such that \(w:(0,1)\times \Omega \rightarrow \Lambda ^{1}\) is smooth and \(s\rightarrow f_{s}\) are paths in \(\mathcal {H}\) that start at \(\bar{f}_{0}\) and end at \(\bar{f} _{1}.\) Unlike (1.9), (1.13) is not a convex minimization problem and so, it is not known to have minimizers. However, if a minimizer (fA) of problem (1.9) is such that \(f_{s}\) is non-degenerate for almost every \(s\in (0,1)\), then \((f,v):=(f,A\,\lrcorner \,f^{-1})\) is a minimizer in (1.13).

There is a sharp contrast between the search of optimal paths in the set of closed k-forms, when \(1\le k\le n-1,\) and that of the case \(k=n\). This, can well be illustrated by comparing the case \(k=2,\) expressed in terms of electro-magnetism, to the case \(k=n,\) expressed as a mass transport problem. Consider a bounded open convex (or contractible) set \(O\subset \mathbb {R}^{3}\) and set

$$\begin{aligned} \Omega :=(0,T)\times O. \end{aligned}$$

Define \(\mathcal {S}\) to be the set of pairs of electro/magnetic time dependent vector fields

$$\begin{aligned} (B,E):(0,T)\times O\rightarrow \mathbb {R}^{6} \end{aligned}$$

which are integrable, satisfy a certain boundary conditions [omitted now but formulated in Subsection E to match (1.1)] and satisfy Gauss’s law for magnetism and the Maxwell–Faraday induction equations

$$\begin{aligned} \nabla \cdot B=0,\quad \partial _{t}B+\nabla \times E=0. \end{aligned}$$
(1.14)

When \(k=2\), (1.8) is equivalent to the search of paths of minimal actions on \(\mathcal {S}\) (cf. Subsection E). Any starting (resp. ending) point \((\bar{B}_{0},\bar{E}_{0})\) (resp. \((\bar{B} _{1},\bar{E}_{1})\)) in \(\mathcal {S}\) is identified with a starting (resp. ending) point \(\bar{f}_{0}\) (resp. \(\bar{f}_{1}\)) in the set of closed 2-forms. Similarly, a path \(s\in [0,1]\rightarrow (B(s),E(s))\) which interpolates between \((B_{0},E_{0})\) and \((B_{1},E_{1}),\) corresponds to a path \(s\in [0,1]\rightarrow f(s),\) lying in the set of closed 2-forms \(\mathcal {H}\), which interpolates between \(\bar{f}_{0}\) and \(\bar{f}_{1}\). If f(s) is not degenerate then there exists \(w:(0,1)\times \Omega \rightarrow \Lambda ^{1}\) such that \(\partial _{s}f+\mathcal {L}_{w}f=0\). Here, it is worth stressing that in contrast with the study of n-forms (i.e. volume forms), intensively studied in the past few years in the theory of optimal transportation, s does not represent a time variable. In the theory of optimal transportation, given two volume forms \(\bar{\mu }_{0}\) and \(\bar{\mu }_{1}\) of same mass, we want to minimize an action over the set of paths \(t\rightarrow \mu (t)\) which interpolate between \(\bar{\mu }_{0}\) and \(\bar{\mu }_{1}.\) For each path \(t\rightarrow \mu (t)\), there exists a velocity vector field v such that the continuity equation \(\partial _{t}\mu +\mathcal {L}_{v}\mu =0\) is satisfied. The action to minimize is an integral over the set of time of an expression either written in terms of \((\mu (t),v(t))\) or equivalently in terms of \((\mu (t),A(t))=(\mu (t),\mu (t)v(t))\). In the case of 2-forms, the time t appears in (1.14) to ensure that f(s) is a closed form for each s, but w is not the physical velocity. Now, the action to be minimized is an integral over the set of parameters s,  of an expression which depends on either (f(s), w(s)) (cf. 1.13) or equivalently \((f(s),A(s))=(f(s),w(s)\,\lrcorner \,f(s))\) (cf. 1.9).

This manuscript contributes to the identification of a non-trivial class of metrics on set of closed k-differential forms, with potential impacts on the study of evolutive equations on the set of closed k-differential forms. The non-homogeneous costs allow for a much richer class of metrics, but come at the expense of yielding transportation problems for which the subdifferentials of the actions are not easily characterized. We then face the study of dual problems which involve k-differential forms, whose differential are not a-priory locally summable. This means that unlike the case when \(k=n\), a difficulty we have to deal with when \(k<n\), is to face a dual problem involving functions for which not all partial derivatives are summable. This means we cannot rely on any classical Sobolev type inequality and need to prove a result such as Lemma 4.7. In this Lemma, we show that up to a translation in one-dimensional interpolation variables, any path on the set of measures of k-differential forms, is controlled by its derivative with respect to the interpolation variable and the \(L^r\)-norm of its co-differential. The point is that we obtain an inequality which does not need to involve the \(L^r\)-norms of both the differential and the co-differential of our k-forms. The proof of the Lemma relies on the use of a subtle Gaffney type inequality and the result is central to obtain needed coercitivity properties of a functional we study in a dual problem. An extremely challenging problem we leave open and which we hope to be the purpose of future investigations, is the regularity properties of geodesics of minimal length. Problem A.2 comments on a systems of PDEs induced by these geodesics.

This manuscript is divided into two parts, the first one containing our central results. The second part is an appendix consisting of examples and technical regularization Lemmas, needed to circumvent the lack of smoothness property of the functions we are dealing with. The appendix ends with a section alluding to the interpretation of our work in the context of electromagnetism.

In Sect. 3, we consider cost functions c on \(\Lambda ^{k}\times \Lambda ^{k-1}\) which assume only finite values, are smooth, strictly convex, with a polynomial growth at infinity. We do not impose that \(c(\omega ,\cdot )\) is p-homogeneous and use standard methods to characterize the subdifferential of the actions along paths of minimal length. This Section will later be useful when studying cost functions which take infinite value. Section 4 is a preliminary section which deals with paths of bounded variations on metric spaces, the metric space in our case being the set of k-currents. We later use these to study Finsler type metrics on the set of k-forms. In Sect. 5 not only the set where c assumes the value \(+\infty \)) is not empty but also \(c^{*},\) the dual of c,  is assumed to have a lower bound which may be linear: \(c^{*}(b,B)\ge \gamma _{6}\left( |b|+|B|^{r} \right) \). This creates a difficulty, usually not faced in the optimal transportation theory, which led to incorporating the two lengthy Sections C and D. We identify and exploit a dual maximization problem to characterize the paths minimizing our action. When \(k=n\), in the dual problem, all the partial derivatives of a scalar function are controlled. When \(k<n\) we face serious technical difficulties since the control of the co-differential of a \((k+1)\)-forms is equivalent to the control of some directional derivatives. We anticipate that the level of complications will substantially increase if we extend the class of cost functions c to include those which are polyconvex or even quasiconvex in a sense to be specified. These considerations, which constitute a new type of challenges, will be addressed in a forthcoming paper [8]. We close our description by drawing the attention of the reader to a recent paper by Brenier and Duan [1], one of the very few related to our context, which considers gradient flows of entropy functionals on the set of differential forms.

Throughout the manuscript, it would have been sufficient to assume that \(\Omega \) is a contractible domain of smooth boundary and not necessarily a convex set. In order to reduce the level of technicality, we chose not to state some of our results under the sharpest assumptions.

2 Preliminaries for the smooth case

For simplicity, throughout the manuscript, \(\Omega \subset \mathbb {R}^{n}\) is assumed to be an open bounded convex set and \(\nu \) denote the outward unit normal to \(\partial \Omega .\) Let \(1 \le k \le n\) be an integer. We assume that \(r, p \in (1, \infty )\) are conjugate of each other in the sense that \(r+p=rp.\)

Definition 2.1

Let \(f\in L^{1}\left( \Omega ;\Lambda ^{k}\right) \), let \(A\in L^{1}\left( \Omega ;\Lambda ^{k-1}\right) \) and \(B\in L^{1}\left( \Omega ;\Lambda ^{k+1}\right) \).

  1. (i)

    We write \(-df=A\) (resp. \(-\delta f=B\)) in \(\Omega \) in the weak sense if for any \(h\in C_{c}^{\infty }\left( \Omega ;\Lambda ^{k}\right) \)

    $$\begin{aligned} \int _{\Omega }\left\langle f;h\right\rangle =\int _{\Omega }\left\langle A;\delta h\right\rangle \qquad \left( \hbox {resp.}\;\int _{\Omega }\left\langle f;h\right\rangle =\int _{\Omega }\left\langle B;dh\right\rangle \right) . \end{aligned}$$
  2. (ii)

    Similarly if we want to express in the weak sense

    $$\begin{aligned} (i) \left\{ \begin{array}{ll} -dA=f &{} \text {in }\Omega \\ \nu \wedge A=0 &{} \text {on }\partial \Omega \end{array} \right. \quad \left( \text {resp.} \quad (ii) \left\{ \begin{array}{ll} -\delta B=g &{} \text {in }\Omega \\ \nu \,\lrcorner \,B=0 &{} \text {on }\partial \Omega \end{array} \right. \right) , \end{aligned}$$
    (2.1)

    we impose that for any \(h\in C^{\infty }\left( \bar{\Omega };\Lambda ^{k}\right) \)

    $$\begin{aligned} \int _{\Omega }\left\langle f; h\right\rangle =\int _{\Omega }\left\langle A ;\delta h\right\rangle \qquad \left( \hbox {resp.}\; \int _{\Omega }\left\langle f; h\right\rangle =\int _{\Omega }\left\langle B; dh\right\rangle \right) . \end{aligned}$$
  3. (iii)

    We say that f is in the weak sense a closed (resp. co-closed) differential form if \(df=0\) (resp. \(\delta f=0\)) in \(\Omega \).

We consider k-forms \(\bar{f}_{0},\bar{f}_{1}\in L^{p}\left( \Omega ;\Lambda ^{k}\right) \) such that, if \(1\le k\le n-1,\)

$$\begin{aligned} \left\{ \begin{array}{ll} d(\bar{f}_{1}-\bar{f}_{0})\equiv 0 &{} \quad \text {in the weak sense of in} \;\Omega \\ (\bar{f}_{1}-\bar{f}_{0})\wedge \nu =0 &{}\quad \text {in the weak sense on} \;\partial \Omega \end{array} \right. \end{aligned}$$
(2.2)

and, if \(k=n,\)

$$\begin{aligned} \int _{\Omega }(\bar{f}_{1}-\bar{f}_{0} )dx=0. \end{aligned}$$
(2.3)

Definition 2.2

We say that \((f, A) \in P^{p}(\bar{f}_{0}, \bar{f}_{1})\) if

$$\begin{aligned} f \in L^{p}\left( (0,1) \times \Omega ; \Lambda ^{k} \right) , \quad A \in L^{p}\left( (0,1) \times \Omega ; \Lambda ^{k-1} \right) \end{aligned}$$

and

$$\begin{aligned} \int _{0}^{1} ds \int _{\Omega }\left( \langle \partial _{s} h ; f \rangle + \langle \delta h; A \rangle \right) dx= \int _{\Omega }\left( \langle h_{1} ; \bar{f}_{1} \rangle -\langle h_{0} ; \bar{f}_{0} \rangle \right) dx \end{aligned}$$
(2.4)

for all \(h \in C^{1} \left( [0,1] \times \bar{\Omega }; \Lambda ^{k} \right) .\)

Remark 2.3

Assume (2.2) holds when \(1 \le k\le n-1\) and (2.3) holds when \(k=n.\)

  1. (i)

    By Theorem 7.2 [7], there exists in the weak sense, \(\bar{A}\in W^{1,p}\left( \Omega ;\Lambda ^{k-1}\right) \) satisfying

    $$\begin{aligned} \left\{ \begin{array}{ll} d\bar{A}+\bar{f}_{1}-\bar{f}_{0}=0 &{} \quad \delta \bar{A}=0\quad \text {in } \Omega \\ \nu \wedge \bar{A}=0 &{} \text {on }\partial \Omega \end{array} \right. \end{aligned}$$

    and there exists a constant \(C=C\left( \Omega ,p,k\right) \) such that

    $$\begin{aligned} ||\bar{A}||_{W^{1,p}\left( \Omega ;\Lambda ^{k-1}\right) }\le C||f||_{L^{p} }. \end{aligned}$$
  2. (ii)

    We have \((\bar{f}_{s},\bar{S}_{s}):=\left( (1-s)\bar{f}_{0} +s\bar{f}_{1},\bar{A}\right) \in P^{p}(\bar{f}_{0},\bar{f}_{1})\).

Definition 2.4

We define \(\mathbf {B}^{r}\left( (0,1) \times \Omega ; \Lambda ^{k} \right) \) to be the set of h such that

$$\begin{aligned} h,\; \partial _{s} h \in L^{r} \left( (0,1) \times \Omega ; \Lambda ^{k} \right) \end{aligned}$$

and there exists

$$\begin{aligned} B \in L^{r} \left( (0,1) \times \Omega ; \Lambda ^{k-1} \right) \end{aligned}$$

such that

$$\begin{aligned} \int _{0}^{1} ds \int _{\Omega }\langle h ; d\psi \rangle dx= -\int _{0}^{1} ds \int _{\Omega }\langle B ; \psi \rangle dx \quad \forall \; \psi \in C_{c}^{1} \left( (0,1) \times \Omega ; \Lambda ^{k-1} \right) . \end{aligned}$$
(2.5)

Here, \(\partial _{s} h\) is the distributional derivative of h with respect to s.

2.1 A weak time continuity property for \(P^{p}(\bar{f}_{0}, \bar{f}_{1})\)

Let \((f, A) \in P^{p}(\bar{f}_{0}, \bar{f}_{1}).\) By Fubini’s theorem, the function \(s \rightarrow \int _{\Omega }|f(s,x)|^{p} dx\) is in \(L^{1}(0,1)\) and so, its Lebesgue points are of full measure in (0, 1). If \(\phi \in C^{1}(\bar{\Omega })\) we set

$$\begin{aligned} L(s,f, \phi )=\int _{\Omega }\langle f(s,x) ; \phi (x) \rangle dx. \end{aligned}$$

Using \(h(s,x)=\alpha (s) \phi (x)\) in (2.4) for arbitrary \(\alpha \in C^{1}([0,1])\), we obtain that there is a set \(\mathcal {N}_{\phi }\) of null Lebesgue measure such that \(L(\cdot , f, \phi )\) coincides on \((0,1) {\setminus }\mathcal {N}_{\phi }\) with a function \(\overline{ L(\cdot , f, \phi )} \in W^{1,p}(0,1).\) More precisely,

$$\begin{aligned} \overline{L(\cdot ,f, \phi )}(s)= \lim _{\delta \rightarrow 0^{+}} {\frac{1 }{\delta }} \int _{s}^{s+\delta } L(\tau , f, \phi ) d\tau . \end{aligned}$$

The distributional derivative of \(L(\cdot , f, \phi )\) is

$$\begin{aligned} \partial _{s} L(\cdot , f, \phi )= \int _{\Omega }\langle A(s,x) ; \delta \phi (x) \rangle dx \end{aligned}$$
(2.6)

We have the following Lemma.

Lemma 2.5

There exists a function \(\tilde{f} \in L^{p}\left( (0,1) \times \Omega ; \Lambda ^{k} \right) \) such that the following hold.

  1. (i)

    \(\tilde{f} = f\) for almost every \((0,1) \times \Omega \)

  2. (ii)

    For any \(\phi \in C_{c}^{1}(\Omega ; \Lambda ^{k})\), \(\overline{L(\cdot , f, \phi )}= L(\cdot , \tilde{f}, \phi )\) everywhere on (0, 1). In particular, \(L(\cdot , \tilde{f}, \phi ) \in W^{1,p}(0,1)\) is continuous.

Remark 2.6

Thanks to Lemma 2.5, we will always tacitly assume that given \((f, A) \in P^{p}(\bar{f}_{0}, \bar{f}_{1})\) then for any \(\phi \in C_{c}^{1}(\Omega ; \Lambda ^{k})\), \(L(\cdot , f, \phi ) \in W^{1,p}(0,1)\) is continuous.

2.2 Properties of \(\mathbf {B}^{r}\left( (0,1) \times \Omega ; \Lambda ^{k} \right) \)

Lemma 2.7

If \(h\in \mathbf {B}^{r}\left( (0,1)\times \Omega ;\Lambda ^{k}\right) ,\) then for \(\mathcal {L}^{1}\)-almost every \(s\in (0,1)\) we have \(B(s,\cdot )\in L^{r}(\Omega )\) and \(B(s,\cdot )=\delta h(s,\cdot )\) is the weak sense.

Proof

Observe first that by Fubini’s theorem,

$$\begin{aligned} B \in L^{r}\left( (0,1) \times \Omega ; \Lambda ^{k-1} \right) \quad \implies \quad B(s, \cdot ) \in L^{r}(\Omega ; \Lambda ^{k-1}) \quad \mathcal {L}^{1} -\hbox {a. e. on} \quad (0,1). \end{aligned}$$

Let \(\{ g_{i}\}_{i=1}^{\infty }\subset C_{c}^{1}(\Omega )\) be a dense subset of \(L^{p}(\Omega ).\) If for \(w \in C_{c}^{1}(0,1)\) we set \(\psi (s,x)=w(s) g_{i}(x)\) then (2.5) reads off

$$\begin{aligned} \int _{0}^{1} w(s) ds \int _{\Omega }\langle h ; dg_{i} \rangle dx= -\int _{0}^{1} w(s) ds \int _{\Omega }\langle B(s, \cdot ) ; g_{i} \rangle dx. \end{aligned}$$

Thus, there exists a set \(N_{i} \subset (0,1)\) of \(\mathcal {L}^{1}\)-null measure such that

$$\begin{aligned} \int _{\Omega }\langle h ; dg_{i} \rangle dx= - \int _{\Omega }\langle B(s, \cdot ) ; g_{i} \rangle dx \end{aligned}$$
(2.7)

for any \(s \in (0,1) {\setminus } N_{i}\). Thus, (2.7) hold for all \(s \in (0,1) {\setminus } N\) if N is the union of the \(N_{i}\)’s. We conclude that

$$\begin{aligned} \int _{\Omega }\langle h ; dg \rangle dx= - \int _{\Omega }\langle B(s, \cdot ) ; g \rangle dx \end{aligned}$$

for any \(s \in (0,1) {\setminus } N\) and any \(g \in C_{c}^{1}(\Omega )\). This concludes the proof of the Lemma. \(\square \)

Remark 2.8

By standard approximation results, it is enough to assume that \(\Omega \) is an open bounded contractible set of locally Lipschitz boundary \(\partial \Omega \) to obtain that if \((f, A) \in P^{p}(\bar{f}_{0}, \bar{f}_{1})\) then (2.4) holds for \(h \in W^{1,r}\left( (0,1) \times \Omega ; \Lambda ^{k}\right) \). The proof of the following Lemma, which extends (2.4) to \(h\in \mathbf {B}^{r}\left( (0,1) \times \Omega ; \Lambda ^{k} \right) \), can be obtained by standard methods.

Lemma 2.9

If \((f,A)\in P^{p}(\bar{f}_{0},\bar{f}_{1})\) and \(h\in \mathbf {B}^{r}\left( (0,1)\times \Omega ;\Lambda ^{k}\right) ,\) then (2.4) holds.

Corollary 2.10

(An invariant) If \((f,A)\in P^{p}(\bar{f}_{0},\bar{f} _{1})\) and \(h\in \mathbf {B}^{r}\left( (0,1)\times \Omega ;\Lambda ^{k}\right) ,\) then

$$\begin{aligned} \int _{0}^{1}ds\int _{\Omega }\left( \langle \partial _{s}h;f\rangle +\langle \delta h;A\rangle \right) dx=\int _{0}^{1}ds\int _{\Omega }\left( \langle \partial _{s} h;\bar{f}\rangle +\langle \delta h;\bar{A}\rangle \right) dx. \end{aligned}$$

Indeed, by Lemma 2.9 these expressions depend only on the initial and final values of h and f.

3 Duality results for smooth superlinear integrands of finite values

Let \(p,r\in (1,\infty )\) be such that \(rp=r+p\) and let \(\bar{f}_{0},\bar{f}_{1}\in L^{p}\left( \Omega ;\Lambda ^{k}\right) \) be two k-forms such that, in the weak sense (2.2) holds when \(1\le k\le n-1\) and (2.3) holds when \(k=n.\) Let

$$\begin{aligned} c:\Lambda ^{k}\times \Lambda ^{k-1}\rightarrow \mathbb {R},\quad c^{*} :\Lambda ^{k}\times \Lambda ^{k-1}\rightarrow (-\infty ,\infty ] \end{aligned}$$

where c is convex and \(c^{*}\) is the Legendre transform of c

$$\begin{aligned} \inf c>-\infty \end{aligned}$$
(3.1)

and

$$\begin{aligned} c^{*}(b,B)\ge \gamma _{1}\left( |b|^{r}+|B|^{r}\right) -\gamma _{2} =:E(b,B) \end{aligned}$$
(3.2)

for any \(b\in \Lambda ^{k}\) and \(B\in \Lambda ^{k-1}.\) Here, \(\gamma _{1} ,\gamma _{2}>0\) are prescribed constants.

Remark 3.1

Since the Legendre transform reverses order, the following hold.

  1. (i)

    If \(c^{*}\) satisfies (3.2) then for any \(\omega \in \Lambda ^{k}\) and \(\xi \in \Lambda ^{k-1}\)

    $$\begin{aligned} c(\omega ,\xi ) \le E^{*}(\omega ,\xi )= \gamma _{2}+ \gamma _{1}(r-1) {\frac{|\omega |^{p}+ |\xi |^{p} }{(r\gamma _{1})^{p}}}. \end{aligned}$$
  2. (ii)

    Similarly, assume there are constants \(\gamma _{6},\gamma _{7}>0\) such that for any \((\omega ,\xi )\in \Lambda ^{k}\times \Lambda ^{k-1}\) we have

    $$\begin{aligned} c(\omega ,\xi )\ge \gamma _{6}(|\omega |^{p}+|\xi |^{p})-\gamma _{7} . \end{aligned}$$
    (3.3)

    Then for any \(b\in \Lambda ^{k}\) and \(B\in \Lambda ^{k-1}\)

    $$\begin{aligned} c^{*}(b,B)\le \gamma _{7}+\gamma _{6}(p-1){\frac{|b|^{r}+|B|^{r}}{(p\gamma _{6})^{r}}}. \end{aligned}$$
  3. (iii)

    If (3.1) holds then \(c^{*}(0,0)=-\inf c\) is a finite real number.

We define \(\mathcal {C}: \Lambda ^{k} \times \Lambda ^{k-1} \rightarrow (-\infty , \infty ]\) by

$$\begin{aligned} \mathcal {C}(f, A)= \int _{(0,1) \times \Omega } c(f, A) ds dx \end{aligned}$$

for

$$\begin{aligned} (f, A) \in L^{p}\left( (0,1) \times \Omega ; \Lambda ^{k} \right) \times L^{p}\left( (0,1) \times \Omega ; \Lambda ^{k-1} \right) \end{aligned}$$

The following proposition is obtained using standard techniques of the direct methods of the calculus of variations.

Proposition 3.2

Suppose \(\bar{f}_{0}, \bar{f}_{1}\in L^{p}\left( \Omega ;\Lambda ^{k}\right) \) are k-forms such that (2.2) holds when \(1\le k\le n-1\) and (2.3) holds when \(k=n.\) Suppose \(c: \Lambda ^{k} \times \Lambda ^{k-1} \rightarrow (-\infty , \infty ]\) is convex, lower semicontinuous and satisfies (3.3). Then there exists \((f^{*},A^{*})\) that minimizes \(\mathcal {C}\) over \(P^{p}(\bar{f}_{0}, \bar{f}_{1}).\)

For \(h \in \mathbf {B}^{r}\left( (0,1) \times \Omega ; \Lambda ^{k} \right) \) we set

$$\begin{aligned} \mathcal {D}(h): =\int _{\Omega } \left( \left\langle \bar{f}_{1};h_{1} \right\rangle -\left\langle \bar{f}_{0};h_{0}\right\rangle \right) dx -\int _{(0,1) \times \Omega } c^{*}\left( \partial _{s} h, \delta h\right) dsdx, \end{aligned}$$

and for \(s \in [0,1]\) set

$$\begin{aligned} \bar{f}_{s}=(1-s)\bar{f}_{0}+s\bar{f}_{1}, \quad \bar{A}_{s}:= \bar{A}, \end{aligned}$$

where \(\bar{A}\) is given by Remark 2.3 (i). By Remark 2.3 (ii), \((\bar{f}, \bar{A}) \in P^{p}(\bar{f}_{0}, \bar{f}_{1})\) and so,

$$\begin{aligned} \mathcal {D}(h) = \int _{(0,1) \times \Omega } \left( \left\langle \bar{A} ; \delta h\right\rangle +\left\langle \bar{f}; \partial _{s} h\right\rangle - c^{*}\left( \partial _{s} h, \delta h\right) \right) dsdx. \end{aligned}$$
(3.4)

Thus, \(\mathcal {D}(h)\) depends only on \(\partial _{s} h\) and \(\delta h\).

Remark 3.3

Assume \(c^{*}\) satisfies (3.2). Then

  1. (i)

    There exist constant \(\gamma _{4}, \gamma _{5}>0\) which depends only on \(\Omega \), \(||\bar{f}_{0}||_{p}\), \(||\bar{f}_{1}||_{p}\) \(\gamma _{1}\), \(\gamma _{2}\), s and r such that

    $$\begin{aligned} \mathcal {D}(h) \le \gamma _{5}- \gamma _{4} \left( ||\delta h||_{r}^{r}+||\partial _{s} h||_{r}^{r} \right) . \end{aligned}$$
    (3.5)
  2. (ii)

    There exists a constant C depending only on \(\Omega \), k and r such that for any \(h \in \mathbf {B}^{r}\left( (0,1) \times \Omega ; \Lambda ^{k} \right) \) there is \(\bar{h} \in \mathbf {B}^{r}\left( (0,1) \times \Omega ; \Lambda ^{k} \right) \) such that \(\mathcal {D}(h)=\mathcal {D}(\bar{h})\) and

    $$\begin{aligned} ||\bar{h}(s, \cdot )||_{L^{r}(\Omega )} \le C ||\delta \bar{h} ||_{r} + ||\partial _{s} \bar{h}||_{r} \quad \mathcal {L}^{1} - \hbox {a.e. on} \;\; (0,1). \end{aligned}$$
  3. (iii)

    If c satisfies (3.1) then \(\mathcal {D} (0)>-\infty .\)

Proof

(i) Using the expression of \(\mathcal {D}\) in (3.4), we have

$$\begin{aligned} \mathcal {D}(h) \le ||\bar{A}||_{p} ||\delta h||_{r}+||\bar{f}||_{p} ||\partial _{s} h||_{r} +\gamma _{2}+ \gamma _{1} \left( \mathcal {L}^{d} (\Omega )-||\partial _{s} h||_{r}^{r}-||\delta h||_{r}^{r} \right) . \end{aligned}$$

This, yields (i).

(ii) By Lemma 2.7 there exists \(t_{0} \in (0,1)\) such that

$$\begin{aligned} \delta h(t_{0}, \cdot )=B(t_{0}, \cdot ), \quad ||\delta h(t_{0}, \cdot )||_{L^{r}(\Omega )}^{r} \le {\frac{||\delta h||_{r}^{r} }{\mathcal {L}^{d}(\Omega )}}. \end{aligned}$$
(3.6)

By Theorems 7.2 and 7.4 [7] (written for \(r \in [2,\infty )\) but extendable to \(r \in (1,2)\)) there is \(\bar{h}_{t_{0}} \in W^{1,r}(\Omega ; \Lambda ^{k})\) such that

$$\begin{aligned} \left\{ \begin{array}{ll} \delta \bar{h}_{t_{0}}= \delta h(t_{0}, \cdot ), \;\; d \bar{h}_{t_{0}} =0 &{}\quad \text {in }\quad \Omega \\ \nu \wedge \bar{h}_{t_{0}}=0 &{}\quad \text {on }\partial \Omega . \end{array} \right. \end{aligned}$$
(3.7)

Furthermore, there is a constant C which depends only on \(\Omega \), k and r such that

$$\begin{aligned} \Vert \bar{h}_{t_{0}} \Vert _{W^{1,r}}\le C (\mathcal {L}^{d}(\Omega ))^{\frac{1 }{r}} \Vert \delta h(t_{0}, \cdot ) \Vert _{L^{r}(\Omega )}. \end{aligned}$$

This, together with (3.6) implies

$$\begin{aligned} \Vert \bar{h}_{t_{0}} \Vert _{W^{1,r}}\le C ||\delta h ||_{r}. \end{aligned}$$
(3.8)

Define

$$\begin{aligned} \bar{h}(s, x)=h(s,x)-h(t_{0}, x)+ \bar{h}_{t_{0}}(x). \end{aligned}$$

We have

$$\begin{aligned} \bar{h}(s, \cdot )= \bar{h}_{t_{0}} + \int _{t_{0}}^{s} \partial _{s} \bar{h}(\tau , \cdot ) d\tau = \bar{h}_{t_{0}} + \int _{t_{0}}^{s} \partial _{s} h(\tau , \cdot ) d\tau . \end{aligned}$$

Thus,

$$\begin{aligned} ||\bar{h}(s, \cdot ) -\bar{h}_{t_{0}}||^{r}_{L^{r}(\Omega )} = \int _{\Omega }\left| \int _{t_{0}}^{s} \partial _{s} h(\tau , x) d\tau \right| ^{r} dx\le ||\partial _{s} h||^{r}_{r}. \end{aligned}$$

This, together with (3.8) yields

$$\begin{aligned} ||\bar{h}(s, \cdot )||_{L^{r}(\Omega )} \le ||\bar{h}_{t_{0}}||_{L^{r}(\Omega )}+ ||\partial _{s} h||_{r} \le C ||\delta h ||_{r} + ||\partial _{s} h||_{r}. \end{aligned}$$

Note that \(\partial _{s} h=\partial _{s} \bar{h}\), \(\delta h=\delta \bar{h}\) to conclude the proof of (ii).

(iii) Since \(\mathcal {D}(0)=-\mathcal {L}^{d}(\Omega ) c^{*}(0, 0)\) and by Remark 3.1, \(c^{*}(0, 0)\) is finite we obtain (iii). \(\square \)

We will often refer to the following proposition, which can be obtained using standard techniques of the direct methods of the calculus of variations.

Proposition 3.4

Assume c satisfies (3.1), \(c^{*}\) satisfies (3.2), \((f, A) \in P^{p}(\bar{f}_{0}, \bar{f}_{1})\) and \(h \in \mathbf {B}^{r}\left( (0,1) \times \Omega ; \Lambda ^{k} \right) \). Then

  1. (i)

    \(\mathcal {C}(f, A) \ge \mathcal {D}(h).\)

  2. (ii)

    \(\mathcal {C}(f, A) = \mathcal {D}(h)\) if and only if \((f , A) \in \partial _{\cdot }c^{*}(\partial _{s} h , \delta h)\) for almost every \((s, x) \in (0,1) \times \Omega .\)

Set

$$\begin{aligned} c_{\epsilon }(f,A):= c(f,A)+{\frac{\epsilon }{p}}(|f|^{p}+|A|^{p}), \qquad \forall \; (f, A) \in \Lambda ^{k} \times \Lambda ^{k-1}. \end{aligned}$$

Set

$$\begin{aligned} \mathcal {D}_{\epsilon }(h):= \int _{(0,1) \times \Omega } \left( \left\langle \bar{A} ; \delta h\right\rangle +\left\langle \bar{f}; \partial _{s} h\right\rangle - c^{*}_{\epsilon }\left( \partial _{s} h, \delta h\right) \right) ds dx \end{aligned}$$
(3.9)

and

$$\begin{aligned} \mathcal {C}_{\epsilon }(f, A):= \int _{(0,1) \times \Omega } c_{\epsilon }\left( f, A\right) ) ds dx. \end{aligned}$$

We now record a remark on convex analysis, which is found in classical literature on the topic.

Remark 3.5

Suppose \(c^{*}\) satisfies (3.2) and \(\epsilon \in (0,1)\).

  1. (i)

    There exist \(\gamma _{1}^{*}, \gamma _{2}^{*}>0\) independent of \(\epsilon \) such that

    $$\begin{aligned} c_{\epsilon }^{*}(b, B) \ge \gamma _{1}^{*} \left( |b|^{r}+|B|^{r} \right) -\gamma _{2}^{*} \end{aligned}$$
  2. (ii)

    We have that \(c^{*}_{\epsilon }\) is of class \(C^{1}\) and its domain is \(\Lambda ^{k} \times \Lambda ^{k-1}\) and

    $$\begin{aligned} c^{*}_{\epsilon }\in C^{1}\left( \Lambda ^{k} \times \Lambda ^{k-1} \right) . \end{aligned}$$
  3. (iii)

    There exists a constant \(C_{\epsilon }\) such that

    $$\begin{aligned} |\nabla c_{\epsilon }^{*}(b, B)| \le C_{\epsilon }\left( |b|^{r-1} +|B|^{r-1}+1 \right) . \end{aligned}$$

Lemma 3.6

(Relying on the smoothness of \(c_{\epsilon }\) to compute the differential of the action) Assume \(c^{*}\) satisfies (3.2) and \(\epsilon \in (0,1).\) Let \(h^{*}, h \in \mathbf {B}^{r}\left( (0,1) \times \Omega ; \Lambda ^{k} \right) \) and set \(N (u)= \mathcal {D}_{\epsilon }(h^{*}+ u h).\) Then,

$$\begin{aligned} N^{\prime }(0)= \int _{(0,1) \times \Omega } \left\langle \bar{A} - A_{\epsilon }; \delta h\right\rangle ds dx +\int _{(0,1) \times \Omega } \left\langle \bar{f} -f_{\epsilon }; \partial _{s} h\right\rangle ds dx \end{aligned}$$

where

$$\begin{aligned} f_{\epsilon }:=\nabla _{a} c^{*}_{\epsilon }\left( \partial _{s} h^{*}, \delta h^{*} \right) , \quad A_{\epsilon }:= \nabla _{B} c^{*}_{\epsilon }\left( \partial _{s} h^{*}, \delta h^{*}\right) . \end{aligned}$$

Proof

The continuity of \(\nabla c^{*}\) and Remark 3.5 (iii) allow to directly compute \(N^{\prime }(0)\). \(\square \)

Proposition 3.7

(Smoothness of \(c_{\epsilon }\) yields a standard duality result) Suppose c is convex, lower semicontinuous, satisfies (3.1) and \(c^{*}\) satisfies (3.2). Then

  1. (i)

    there exists \(h^{*}\) that maximizes \(\mathcal {D}\) over \(\mathbf {B}^{r}\left( (0,1) \times \Omega ; \Lambda ^{k} \right) \).

  2. (ii)

    there exists \(h_{\epsilon }\) that maximizes \(\mathcal {D}_{\epsilon }\) over \(\mathbf {B}^{r}\left( (0,1) \times \Omega ; \Lambda ^{k} \right) \).

  3. (iii)

    For any \(h \in \mathbf {B}^{r}\left( (0,1) \times \Omega ; \Lambda ^{k} \right) \)

    $$\begin{aligned} \int _{(0,1) \times \Omega } \left\langle \bar{A} - A_{\epsilon }; \delta h\right\rangle ds dx +\int _{(0,1) \times \Omega } \left\langle \bar{f} -f_{\epsilon }; \partial _{s} h\right\rangle ds dx=0. \end{aligned}$$

    where

    $$\begin{aligned} f_{\epsilon }:=\nabla _{a} c^{*}_{\epsilon }\left( \partial _{s} h_{\epsilon }, \delta h_{\epsilon }\right) , \quad A_{\epsilon }:= \nabla _{B} c^{*}_{\epsilon }\left( \partial _{s} h_{\epsilon }, \delta h_{\epsilon }\right) . \end{aligned}$$
    (3.10)
  4. (iv)

    We may assume without loss of generality that there is a constant C independent of \(\epsilon \) such that we can choose \(h_{\epsilon }\) such that

    $$\begin{aligned} \left\| h_{\epsilon }(s, \cdot )\right\| _{L^{r}(\Omega )} \le C ||\delta h_{\epsilon }||_{r}+ || \partial h_{\epsilon } ||_{r} \end{aligned}$$

Proof

(i) Let \(\bar{A}\) be given by Remark 2.3 and set

$$\begin{aligned} \bar{f}(s,x)=(1-s) \bar{f}_{0}(x)+s \bar{f}_{1}(x). \end{aligned}$$

We have \((\bar{f}, \bar{A}) \in P^{p}(\bar{f}_{0}, \bar{f}_{1})\). The bounds in that Remarks 2.3 (i) and 3.1 (i) imply

$$\begin{aligned} \mathcal {C}(\bar{f}, \bar{A})<\infty . \end{aligned}$$

This, together with Proposition 3.4 implies

$$\begin{aligned} D:= \sup _{h} \left\{ \mathcal {D}(h)\; | \; h \in \mathbf {B}^{r}\left( (0,1) \times \Omega ; \Lambda ^{k} \right) \right\} \le C(\bar{f}, \bar{A})<\infty . \end{aligned}$$

By Remark 3.3 (iii) \(D>-\infty \) and by (i) of the same remark, if \(\gamma \) is a real number then the upper level sets of \(\mathcal {D}\) satisfy

$$\begin{aligned}&\left\{ h \in \mathbf {B}^{r}\left( (0,1) \times \Omega ; \Lambda ^{k} \right) \; | \; \mathcal {D}(h) \ge \gamma \right\} \\&\quad \subset \left\{ h \in \mathbf {B}^{r}\left( (0,1) \times \Omega ; \Lambda ^{k} \right) \; \Big | \; ||\delta h||_{r}^{r} + ||\partial _{s} h||_{r}^{r} \le {\frac{\gamma _{5}-\gamma }{\gamma _{4}}} \right\} . \end{aligned}$$

Combining this with Remark 3.3 (ii) we obtain a maximizing sequence \(\{h_{i}\}_{i}\) of \(\mathcal {D}\) over \(\mathbf {B}^{r}\left( (0,1) \times \Omega ; \Lambda ^{k} \right) \) satisfying

$$\begin{aligned} \sup _{i} || h_{i}||_{r}^{r}+ ||\delta h_{i}||_{r}^{r} + ||\partial _{s} h_{i}||_{r}^{r}<\infty . \end{aligned}$$

Hence, we may extract from \(\{h_{i}\}_{i}\) a subsequence which converges weakly to some \(h^{*}\) in \(L^{r}\left( (0,1) \times \Omega ; \Lambda ^{k} \right) \) and such that \(\{\delta h_{i}\}_{i}\) (resp. \(\{\partial _{s} h_{i}\}_{i}\)) converges weakly to \(\delta h^{*}\) (resp. \(\partial _{s} h^{*}\)) in \(L^{r}\left( (0,1) \times \Omega ; \Lambda ^{k} \right) \). We have \(h^{*} \in \mathbf {B}^{r}\left( (0,1) \times \Omega ; \Lambda ^{k} \right) \).

Recall that by (3.4), \(-\mathcal {D}(h_{i})\) can be expressed as a convex function of \(\partial _{s} h_{i}\) and \(\delta h_{i}.\) Therefore, by standard results of convex analysis

$$\begin{aligned} -D = \liminf _{i \rightarrow \infty } -\mathcal {D}(h_{i}) \ge -\mathcal {D} (h^{*}). \end{aligned}$$

This proves that \(h^{*}\) maximizes \(\mathcal {D}\) over \(\mathbf {B} ^{r}\left( (0,1) \times \Omega ; \Lambda ^{k} \right) \).

(ii) By Remark 3.5 we have all the properties needed to replace \(c^{*}\) by \(c^{*}_{\epsilon }\) in the above proof. The proof of (ii) repeats the arguments used in that of (i) but it is even easier.

(iii) Let \(h \in \mathbf {B}^{r}\left( (0,1) \times \Omega ; \Lambda ^{k} \right) \). The real valued function \(u \in \mathbb {R }\rightarrow N_{\epsilon }(u)= \mathcal {D}_{\epsilon }(h_{\epsilon }+ u h)\) achieves its minimum at 0. Since by Lemma 3.6 N is differentiable at 0, we have \(N^{\prime }_{\epsilon }(0)=0\). This is exactly the identity in (iii).

(iv) Is a direct consequence of Remark 3.3 (ii). \(\square \)

Theorem 3.8

(A duality result not requiring smoothness of c) Suppose c is convex, lower semicontinuous, it satisfies (3.1) and \(c^*\) satisfies (3.2). Further assume there are constants \(\gamma _6, \gamma _7>0\) such that c satisfies (3.3). Then

  1. (i)

    there exists \((f^*, A^*)\) which minimizes \(\mathcal {C}\) over \(P^p(\bar{f}_0, \bar{f}_1)\).

  2. (ii)

    For any \(h^*\) that maximizes \(\mathcal {D}\) over \(\mathbf{B}^r\left( (0,1) \times \Omega ; \Lambda ^k \right) \) we have \(\mathcal {C} (f^*,A^*) =\mathcal {D}(h^*).\)

  3. (iii)

    Let \((f, A) \in P^p(\bar{f}_0, \bar{f}_1).\) Then (fA) minimizes \(\mathcal {A}\) over \(P^p(\bar{f}_0, \bar{f}_1)\) if and only if there exists \(h \in \mathbf{B}^r\left( (0,1) \times \Omega ; \Lambda ^k \right) \) such that \((f, A) \in \partial _\cdot c^*(\partial _s h, \delta h)\) for almost every \((s, x) \in (0,1) \times \Omega .\)

Proof

(i) and (ii) Let \(h_{\epsilon }\) be a maximizer of \(\mathcal {D} _{\epsilon }\) as provided in Proposition 3.7 and let

$$\begin{aligned} (f_{\epsilon }, A_{\epsilon }):=\nabla c^{*}_{\epsilon }(\partial _{s} h_{\epsilon }, \delta h_{\epsilon }). \end{aligned}$$

We combine (iii) of the same proposition with the fact that \((\bar{f}, \bar{A}) \in P^{p}(\bar{f}_{0}, \bar{f}_{1})\) (cf. Remark 2.3 (ii)) to obtain that \((f_{\epsilon }, A_{\epsilon }) \in P^{p}(\bar{f}_{0}, \bar{f}_{1}).\) Proposition 3.4 (ii) implies

$$\begin{aligned} \mathcal {C}(f_{\epsilon }, A_{\epsilon })= \mathcal {D}_{\epsilon }(h_{\epsilon }). \end{aligned}$$

We then use Proposition 3.4 (i) to conclude that \((f_{\epsilon }, A_{\epsilon })\) minimizes \(\mathcal {C}_{\epsilon }\) over \(P^{p}(\bar{f}_{0}, \bar{f}_{1})\).

Since for \(\epsilon \in (0,1)\)

$$\begin{aligned} \gamma _{6} \left( ||f_{\epsilon }||^{p}_{p}+||A_{\epsilon }||^{p}_{p} -\gamma _{7} \mathcal {L}^{d}(\Omega ) \right) \le \mathcal {C}_{\epsilon }(f_{\epsilon }, A_{\epsilon }) \le \mathcal {C}_{\epsilon }(\bar{f}, \bar{A}) \le \mathcal {C} _{1}(\bar{f}, \bar{A}), \end{aligned}$$

we have

$$\begin{aligned} S:= \sup _{\epsilon \in (0,1)} ||f_{\epsilon }||^{p}_{p}+||A_{\epsilon }||^{p}_{p}<\infty . \end{aligned}$$
(3.11)

Also, by Remark 3.5 (i) and the maximality property of \(h_{\epsilon }\)

$$\begin{aligned} \gamma _{2}^{*} \mathcal {L}^{n}(\Omega ) \ge -c^{*}(0,0) \mathcal {L} ^{n}(\Omega ) =-\mathcal {D}_{\epsilon }(0) \ge -\mathcal {D}_{\epsilon }(h_{\epsilon }). \end{aligned}$$

Thus, using (3.4) we have

$$\begin{aligned} \gamma _{2}^{*} \mathcal {L}^{n}(\Omega ) \ge \int _{(0,1) \times \Omega } \left( c^{*}(\partial _{s} h_{\epsilon }, \delta h_{\epsilon }) -\left\langle \bar{A} ; \delta h_{\epsilon }\right\rangle -\left\langle \bar{f}; \partial _{s} h_{\epsilon }\right\rangle \right) ds dx. \end{aligned}$$

We again use Remark 3.5 (i) to obtain

$$\begin{aligned} \gamma _{2}^{*} \mathcal {L}^{n}(\Omega ) \ge \gamma ^{*}_{1} \left( || \partial _{s} h_{\epsilon }||_{r}^{r} + || \delta h_{\epsilon }||_{r}^{r} \right) - || \partial _{s} h_{\epsilon }||_{r} ||\bar{f}||_{p} - || \delta h_{\epsilon }||_{r} ||\bar{A} ||_{p} \end{aligned}$$

and so,

$$\begin{aligned} \sup _{\epsilon \in (0,1)} ||\delta h_{\epsilon }||^{r}_{r}+||\partial _{s} h_{\epsilon }||^{r}_{r}<\infty . \end{aligned}$$

Thus by Remark 3.3 (ii), we may assume without loss of generality that

$$\begin{aligned} \bar{S}:= \sup _{\epsilon \in (0,1)} || h_{\epsilon } ||^{r}_{r}+||\delta h_{\epsilon }||^{r}_{r}+||\partial _{s} h_{\epsilon } ||^{r}_{r}<\infty \end{aligned}$$
(3.12)

By (3.11) there exists a subsequence of \((f_{\epsilon _{l}}, A_{\epsilon _{l}})_{l}\) which converges weakly to some \((f^{*}, A^{*})\) in \(L^{p}\left( (0,1) \times \Omega ; \Lambda ^{k} \right) \times L^{p}\left( (0,1) \times \Omega ; \Lambda ^{k-1} \right) \) as l tends to \(\infty .\) Passing to another subsequence if necessary, thanks to (3.12), we may assume without loss of generality that \((h_{\epsilon _{l}})_{l}\) converges weakly in \(L^{r}\) to some \(h^{*} \in \mathbf {B}^{r}\left( (0,1) \times \Omega ; \Lambda ^{k} \right) \). Thus, \((\delta h_{\epsilon _{l}})_{l}\) converges weakly in \(L^{r}\) to \(\delta h^{*}\) and \((\partial _{s} h_{\epsilon _{l}})_{l}\) converges weakly in \(L^{r}\) to \(\partial _{s} h^{*}.\) Letting \(\epsilon _{l}\) tend to 0 in Proposition 3.7 (iii) we obtain for any \(h \in W^{1,r}\left( (0,1) \times \Omega ; \Lambda ^{k} \right) \)

$$\begin{aligned} \int _{(0,1) \times \Omega } \left\langle \bar{A} - A^{*} ; \delta h\right\rangle ds dx +\int _{(0,1) \times \Omega } \left\langle \bar{f} -f^{*}; \partial _{s} h\right\rangle ds dx=0. \end{aligned}$$

We use the the fact that by Remark 2.3 (ii), \((\bar{f}, \bar{A}) \in P^{p}(\bar{f}_{0}, \bar{f}_{1})\) to conclude that

$$\begin{aligned} \int _{\Omega } \left( \langle \bar{f}_{1} ; h_{1} \rangle -\langle \bar{f}_{0} ; h_{0} \rangle \right) dx - \int _{(0,1) \times \Omega } \left( \left\langle A^{*} ; \delta h\right\rangle - \left\langle f^{*}; \partial _{s} h\right\rangle \right) ds dx=0. \end{aligned}$$

and so, \((f^{*}, A^{*}) \in P^{p}(\bar{f}_{0}, \bar{f}_{1}).\)

We first use the fact that \((f_{\epsilon }, A_{\epsilon }) \in \partial c^{*}_{\epsilon }( \partial _{s} h_{\epsilon }, \delta h_{\epsilon })\) and then use the fact that \(c_{\epsilon }\ge c\) to obtain

$$\begin{aligned} \left\langle A_{\epsilon }; \delta h_{\epsilon } \right\rangle + \left\langle f_{\epsilon }; \partial _{s} h_{\epsilon }\right\rangle =c_{\epsilon }\left( f_{\epsilon }, A_{\epsilon }\right) + c^{*}_{\epsilon }\left( \partial _{s} h_{\epsilon }, \delta h_{\epsilon }\right) \ge c\left( f_{\epsilon }, A_{\epsilon }\right) + c^{*}_{\epsilon }\left( \partial _{s} h_{\epsilon }, \delta h_{\epsilon }\right) . \end{aligned}$$
(3.13)

Also

$$\begin{aligned} c^{*}_{\epsilon }\left( \partial _{s} h_{\epsilon }, \delta h_{\epsilon }\right)&= \left\langle A_{\epsilon }; \delta h_{\epsilon } \right\rangle + \left\langle f_{\epsilon }; \partial _{s} h_{\epsilon } \right\rangle -c\left( f_{\epsilon }, A_{\epsilon }\right) -{\frac{\epsilon }{p}} \left( |f_{\epsilon }|^{p} + |A_{\epsilon }|^{p} \right) \\&\ge c^{*}\left( \partial _{s} h_{\epsilon }, \delta h_{\epsilon }\right) -{\frac{\epsilon }{p}} \left( |f_{\epsilon }|^{p} + |A_{\epsilon }|^{p} \right) . \end{aligned}$$

We combine this with (3.13) to conclude that

$$\begin{aligned} \int _{(0,1) \times \Omega } \left( \left\langle A_{\epsilon }; \delta h_{\epsilon } \right\rangle + \left\langle f_{\epsilon }; \partial _{s} h_{\epsilon }\right\rangle \right) ds dx \ge \int _{(0,1) \times \Omega } \left( c^{*}\left( \partial _{s} h_{\epsilon }, \delta h_{\epsilon }\right) + c\left( f_{\epsilon }, A_{\epsilon }\right) \right) ds dx - {\frac{\epsilon S }{p}}. \end{aligned}$$

Since \((f_{\epsilon }, A_{\epsilon }) \in P^{p}(\bar{f}_{0}, \bar{f}_{1}),\) we may use Remark 2.10 in the previous inequality to obtain

$$\begin{aligned} \int _{(0,1) \times \Omega } \left( \left\langle \bar{A}; \delta h_{\epsilon } \right\rangle + \left\langle \bar{f} ; \partial _{s} h_{\epsilon }\right\rangle \right) ds dx \ge \int _{(0,1) \times \Omega } \left( c^{*}\left( \partial _{s} h_{\epsilon }, \delta h_{\epsilon }\right) + c\left( f_{\epsilon }, A_{\epsilon }\right) \right) ds dx- {\frac{\epsilon S }{p}} \end{aligned}$$
(3.14)

One lets \(\epsilon _{l}\) tend to 0 to derive the inequality

$$\begin{aligned} \int _{(0,1) \times \Omega } \left( \left\langle \bar{A}; \delta h \right\rangle +\left\langle \bar{f} ; \partial _{s} h\right\rangle \right) ds dx \ge \int _{(0,1) \times \Omega } \left( \left\langle \bar{A}; \delta h\right\rangle + \left\langle \bar{f} ; \partial _{s} h\right\rangle \right) ds dx. \end{aligned}$$

This proves that

$$\begin{aligned} \int _{(0,1) \times \Omega } \left( \left\langle \bar{A}; \delta h \right\rangle + \left\langle \bar{f} ; \partial _{s} h\right\rangle \right) ds dx= \int _{(0,1) \times \Omega } \left( c^{*}\left( \partial _{s} h^{*}, \delta h^{*}\right) +c(f^{*}, A^{*}) \right) ds dx. \end{aligned}$$
(3.15)

Rearranging, and using the expression of \(\mathcal {D}\) in (3.4), we have \(\mathcal {D}(h^{*})=\mathcal {C}(f^{*}, A^{*}).\) By Proposition 3.4 (i), \((f^{*}, A^{*})\) minimizes \(\mathcal {C}\) over \(P^{p}(\bar{f}_{0}, \bar{f}_{1})\) and \(h^{*}\) maximizes \(\mathcal {D}\) over \(\mathbf {B}^{r}\left( (0,1) \times \Omega ; \Lambda ^{k} \right) \).

(iii) Let \((f, A) \in P^{p}(\bar{f}_{0}, \bar{f}_{1})\) and \(h \in \mathbf {B} ^{r}\left( (0,1) \times \Omega ; \Lambda ^{k} \right) \). Since \(c(\omega ,\xi ) \ge \gamma _{6} (|\omega |^{p}+|\xi |^{p})-\gamma _{7}\) for all \(\omega \in \Lambda ^{k}\) and \(\xi \in \Lambda ^{k-1}\), there is a constant \(\gamma _{6}^{*}>0\) such that \(c^{*}(b,B) \le \gamma _{6}^{*} (|b|^{r}+|B|^{r})+\gamma _{7}\) for all \(b \in \Lambda ^{k}\) and \(B \in \Lambda ^{k-1}\). This together with the fact that \(c^{*}\) satisfies (3.2) implies \(\mathcal {D} (h)<\infty .\)

By Proposition 3.4, \((f, A) \in \partial c^{*} (\partial _{s} h, \delta h)\) for almost every \((s, x) \in (0,1) \times \Omega \) if and only if h maximizes \(\mathcal {D}\) over \(\mathbf {B}^{r}\left( (0,1) \times \Omega ; \Lambda ^{k} \right) \) and (fA) minimizes \(\mathcal {A}\) over \(P^{p}(\bar{f}_{0}, \bar{f}_{1})\). \(\square \)

4 The set of k-forms: approximations of k-currents

4.1 Notation

Throughout this subsection \(\mathbb {H}\) is a finite dimensional Hilbert space and \(C:\mathbb {H}\rightarrow (-\infty , \infty ]\) is a proper lower semicontinuous convex function. We fix a non empty open bounded convex set \(\Omega \subset {\mathbb {R}}^n\) and \(p \in (1, \infty ).\)

We denote by \(\mathcal {M}(\Omega )\) the set of signed measure of finite total variations. The upper and lower variations \(g^{+}\) and \(g^{-}\) are finite measures and the Jordan decomposition \(g=g^{+}-g^{-}\) holds (cf. e.g. [11]). The total mass of \(|g|:=g^{+}+g^{-}\) is

$$\begin{aligned} ||g||_{\mathcal {M}(\Omega )}=\sup _{f\in C(\bar{\Omega })}\left\{ \int _{\Omega }f(x)g(dx)\;|\;\;|f|\le 1\right\} =\sup _{f\in C_{c}(\Omega )}\left\{ \int _{\Omega }f(x)g(dx)\;|\;\;|f|\le 1\right\} , \end{aligned}$$
(4.1)

\(\left( \mathcal {M}(\Omega ),||\cdot ||\right) \) is a normed space and by the Banach–Alaoglu Theorem, every bounded subset is pre-compact. Thus, \(\left( \mathcal {M}(\Omega ),||\cdot ||\right) \) is a complete space.

Let \(\mathcal {C}\) be a countable dense subset of \(C_{c}(\Omega )\), contained in \(C_{c}^{1}(\Omega )\) and which does not contain the null function. If we denote by \(\hat{\mathcal {C}}\) the set of \(f/||f||_{\infty }\) such that \(f \in \mathcal {C}\) then

$$\begin{aligned} ||g||_{\mathcal {M}(\Omega )}=\sup _{f \in \hat{\mathcal {C}}} \int _{\Omega }f(x) g(dx). \end{aligned}$$
(4.2)

The set of Borel measures with values into \(\Lambda ^{k}\), of finite total mass, will be denoted by \(\mathcal {M}(\Omega ; \Lambda ^{k}).\) This is the set of k-currents of finite mass. For any \(F \in \mathcal {M}(\Omega ; \Lambda ^{k}),\) we define the norm

$$\begin{aligned} ||F||_{\mathcal {M}(\Omega )}= \sup _{G \in C\left( \bar{\Omega };\Lambda ^{k} \right) }\left\{ \int _{\Omega }\langle G(x); F(dx)\rangle \; | \;\;|G(x)| \le 1 \; \forall x \in \Omega \right\} . \end{aligned}$$
(4.3)

Definition 4.1

Given a metric space \((\mathcal {S}, \mathrm {dist})\) the total variation of \(h:[0,1] \rightarrow \mathcal {S}\) is

$$\begin{aligned} \mathrm {TV}(h):= \sup _{m \in \mathbb {N}} \; \sup _{0\le t_{0}< \cdots < t_{m}\le 1} \left\{ \sum _{i=0}^{m-1} \mathrm {dist}\left( h(t_{i}), h(t_{i+1})\right) \right\} . \end{aligned}$$

Definition 4.2

The following definitions can be found respectively in [12, 14]. The recession function of C is \(\bar{C}:\mathbb {H}\rightarrow (-\infty ,\infty ]\) given by

$$\begin{aligned} \bar{C}(v)=\lim _{t\rightarrow \infty }{\frac{C(v_{0}+tv)}{t}}\qquad v\in \mathbb {H}\quad \hbox {where}\quad v_{0}\in \mathbb {H}\;\hbox {is arbitrary.} \end{aligned}$$

One checks that the definition is independent of \(v_{0}.\)

Set

$$\begin{aligned} O:=(0,1) \times \Omega , \qquad z:=(s,x), \qquad dz:=ds dx. \end{aligned}$$

Here, we skip the proof of the following elementary Lemma.

Lemma 4.3

Assume \(g \in L^{p}(O)\) and \(\eta \) be a singular measure. Set \(\eta _{*}:=\eta +\mathcal {L}^{n+1}_{O}\) and let \(E \subset O\) be a Borel set such that

$$\begin{aligned} \eta (O {\setminus } E)= \mathcal {L}^{n+1}(E)=0. \end{aligned}$$
(4.4)

Then for any \(\alpha \in \mathbb {R}\), \(g_{\alpha }:=g(1-\chi _{E}) + \alpha \; \chi _{E}\in L^{p}(O, \eta _{*})\) and \(\mathcal {L}^{n+1}\{g_{\alpha }\not = g \}=0.\)

Remark 4.4

Assume \(c: \Lambda ^{k} \times \Lambda ^{k-1} \rightarrow (-\infty , \infty ]\) is convex, lower semicontinuous and satisfies (5.3). We assume the Legendre transform \(c^{*}: \Lambda ^{k} \times \Lambda ^{k-1} \rightarrow \mathbb {R}\) satisfies (5.4). Let

$$\begin{aligned} b \in \mathcal {M}(O; \Lambda ^{k}), \;\; B \in L^{r}\left( O; \Lambda ^{k-1}\right) . \end{aligned}$$

Let \(b_{s}\) be the singular part of b,  set \(\eta := |b_{s}|\) and let \(E \subset O\) be a Borel set satisfying (4.4). Consider the Radon–Nikodym derivatives \(F:= d b /d\mathcal {L}^{n+1}\) and \(G:= d b_{s} / d \eta .\) Let

$$\begin{aligned} \bar{f} \in L^{p}\left( O; \Lambda ^{k}\right) , \;\; A \in L^{p}\left( O; \Lambda ^{k-1}\right) \end{aligned}$$

be such that

$$\begin{aligned} \int _{O} c(\bar{f}, A) dz<\infty . \end{aligned}$$

Note \(c(\bar{f}, A)\) is finite except may be on a Borel set \(F \subset O\) such that \(\mathcal {L}^{n+1}(F)=0.\) Let \(d_{*}\) be in \(\mathrm {dom}(c).\) According to Lemma 4.3,

$$\begin{aligned} f:=(1-\chi _{E}) \bar{f}+ d_{*} \chi _{E} \in L^{p}\left( O; \Lambda ^{k}, \eta _{*}\right) \end{aligned}$$

where \(\eta _{*}:=\eta + \mathcal {L}^{n+1}|_{O}.\) Furthermore, \(f=\bar{f}\) \(\mathcal {L}^{n+1}\)-almost everywhere

Assume that \(f: O \rightarrow \Lambda ^{k}\) is a Borel map which we are free to modify on a set of null \((\mathcal {L}^{n+1}+|b|)\)-measure. We have

$$\begin{aligned} c(f, A) + c^{*}(F, B) \ge \langle f; F \rangle + \langle A; B \rangle \end{aligned}$$

and so, if \(c(f, A) + c^{*}(F, B) \in L^{1}(O)\) then the positive part of \(\langle f; F \rangle + \langle A; B \rangle \) is of finite Lebesgue integral. In that case, in terms of \(\bar{C}\), the recession function of \(C:=c^{*}\), we have

$$\begin{aligned} \int _{O} c(f, A) dz +\int _{O} c^{*}(b, B) = \int _{O} \left( c(f, A) + c^{*}(F, B) \right) dz+ \int _{O} \overline{c^{*}}(G, 0) d\eta \end{aligned}$$

Since \(c(f, A)<\infty \) \(\eta _{*}\)—a.e., we use Lemma C.1 (i) to infer

$$\begin{aligned} \int _{O} c(f, A) dz +\int _{O} c^{*}(b, B)&\ge \int _{O} \left( \langle f; F \rangle + \langle A; B \rangle \right) dz+ \int _{O} \langle f; G\rangle d\eta \\&= \int _{O} \langle f; b \rangle + \int _{O} \langle A; B \rangle dz. \end{aligned}$$

Equality holds if and only if

$$\begin{aligned} (f, A) \in \partial _{\cdot }c^{*}(F, B) \; \mathcal {L}^{n+1}-\hbox {a.e.} \quad \hbox {and} \quad \overline{c^{*}}(G, 0) = \langle f; G\rangle \quad \eta -\hbox {a.e.} \end{aligned}$$
(4.5)

4.2 Paths of bounded variations on \(\mathcal {M}(\Omega ;\Lambda ^{k})\)

Below, we list results on the trace operator of \(BV\left( (0,1); \mathcal {M}(\Omega ; \Lambda ^{k}) \right) \) functions, needed in the manuscript.

Remark 4.5

There exists a linear bounded trace (explicitely written below as the left/right limits) operator \(T:BV\left( (0,1); \mathcal {M}(\Omega ; \Lambda ^{k}) \right) \rightarrow L^{\infty }\left( \{0,1\}; \mathcal {M}(\Omega ; \Lambda ^{k}) \right) \) such that the following hold for any \(h \in BV\left( (0,1); \mathcal {M}(\Omega ; \Lambda ^{k})\right) \).

  1. (i)

    If h and \(\partial _{s} h\) are continuous on \([0,1] \times \bar{\Omega }\) then

    $$\begin{aligned} T h=h|_{\{0,1\} \times \Omega } \end{aligned}$$
  2. (ii)

    We have the integration by parts formula

    $$\begin{aligned} \int _{0}^{1} ds \int _{\Omega }\langle h(s,dx) ; \partial _{s} g(s,x) \rangle + \int _{(0,1) \times \Omega } \langle \partial _{s} h(ds,dx) ; g(s,x) \rangle = u \end{aligned}$$

    for any \(g \in C^{1}\left( [0,1] \times \bar{\Omega }; \Lambda ^{k} \right) \). Here, we have set

    $$\begin{aligned} u:=\int _{\Omega }\langle T h(1,dx) ; g(1,x) \rangle -\int _{\Omega }\langle T h(0,dx) ; g(0,x) \rangle \end{aligned}$$
  3. (iii)

    If \(h \in BV\left( (0,1); \mathcal {M}(\Omega ; \Lambda ^{k}) \right) \) is such that \(s\rightarrow h(s, \cdot )\) is left continuous at 1 and right continuous at 0 then

    $$\begin{aligned} T h(0, \cdot )= \lim _{s \rightarrow 0^{+}} h(s, \cdot ), \quad T h(1, \cdot )= \lim _{s \rightarrow 1^{-}} h(s, \cdot ) \end{aligned}$$

4.3 Special paths of bounded variations on \(\mathcal {M}(\Omega ;\Lambda ^{k})\)

Let \(h \in L^{1}\left( (0,1); \mathcal {M} (\Omega ;\Lambda ^{k})\right) \) be such that there exists \(b\in \mathcal {M} \left( (0,1) \times \Omega ;\Lambda ^{k}\right) \) such that

$$\begin{aligned} \int _{0}^{1} ds \int _{\Omega }\langle \partial _{s} \psi (s, x); h(s, dx) \rangle = -\int _{(0,1) \times \Omega } \langle \psi (s, x); b(ds, dx) \rangle \end{aligned}$$
(4.6)

for all \(\psi \in C_{c}^{1}\left( (0,1) \times \Omega ;\Lambda ^{k}\right) \). Modifying if necessary, \(h(s, \cdot )\) on a subset of (0, 1) of null Lebesgue (cf. [13]), we always assume without loss of generality that h satisfies the following Lemma.

Lemma 4.6

(A non smooth variant of Remark 3.3(ii)) If (4.6) holds, then for any \(0 \le t_{1}<t_{2}<1\) and \(F \in C_{c}(\Omega ; \Lambda ^{k}),\) we have the following.

  1. (i)
    $$\begin{aligned} \int _{\Omega }\langle F(x); h(t_{2}, dx) \rangle -\int _{\Omega }\langle F(x); h(t_{1}, dx) \rangle = \int _{(t_{1}, t_{2}] \times \Omega } \langle F(x); b(ds, dx) \rangle \end{aligned}$$
  2. (ii)
    $$\begin{aligned} \int _{\Omega }\langle F(x); h(1, dx) \rangle -\int _{\Omega }\langle F(x);h(t_{1}, dx)\rangle = \int _{(t_{1}, 1) \times \Omega } \langle F(x); b(ds, dx)\rangle \end{aligned}$$
  3. (iii)

    Using the definition of \(\mathrm {TV}(h)\) in Definition 4.1 we have

    $$\begin{aligned} \mathrm {TV}(h) \le |b|\left( (0,1) \times \Omega \right) . \end{aligned}$$

Lemma 4.7

Further assume there exists \(B\in L^{r}\left( (0,1);L^{r}(\Omega ;\Lambda ^{k-1})\right) \) such that

$$\begin{aligned} \int _{(0,1)\times \Omega }\langle dg;h\rangle dsdx=-\int _{(0,1)\times \Omega }\langle g;B\rangle dsdx \end{aligned}$$
(4.7)

for all \(g\in C_{c}^{1}\left( (0,1)\times \Omega ;\Lambda ^{k-1}\right) \), we say \(\delta h=B\) in the weak sense and say that \(\delta \phi \) belongs to \(L^{r}\left( (0,1);L^{r}(\Omega ;\Lambda ^{k-1})\right) .\) There exists \(\bar{h}_{t_{0}}\in W^{1,r}(\Omega ;\Lambda ^{k})\) such that if we set \([\bar{h}(s,\cdot ):=h(s,\cdot )-h(t_{0},\cdot )+\bar{h}_{t_{0}}\) then, the following hold.

  1. (i)

    Replacing h by \(\bar{h}\), (4.11) holds for any \(s \in \mathcal {T}\) and any \(H \in C_{c}^{1}(\Omega ;\Lambda ^{k-1})\). In other words, for any \(s \in \mathcal {T}\), we have \(\delta \bar{h}(s, \cdot ) =B(s, \cdot )\).

  2. (ii)

    There exists a constant \(C_{\Omega }\) depending only on \(\Omega \), r and k such that for all \(s\in (0,1)\)

    $$\begin{aligned} ||\bar{h}(s, \cdot )|| \le |b|\left( (0, 1) \times \Omega \right) + C_{\Omega }\mathcal {L}^{n}(\Omega )^{\frac{1 }{r}} \left( \int _{(0,1) \times \Omega } |B(\tau ,x)|^{r} d\tau dx\right) ^{\frac{1 }{r}}. \end{aligned}$$
  3. (iii)

    We have \(\partial _{s} \bar{h} =b\) and \(\delta \bar{h}=B\) in the sense that we may substitute \(\bar{h}\) with h in (4.6) and (4.7).

Proof

By Lemma 4.6, for each \(F \in C^{1}(\Omega ;\Lambda ^{k})\), the real value function

$$\begin{aligned} t \rightarrow \int _{\Omega }\langle F(x); h(s ,dx) \rangle \end{aligned}$$

it is defined everywhere on [0, 1], it is in \(\mathrm {BV}(0,1)\), right continuous on [0, 1) and left continuous at 1. We use (i) of the same Lemma to obtain

$$\begin{aligned} \int _{\Omega }|h|(s, dx) \le \int _{0}^{1} ||h(s, \cdot )||_{\mathcal {M}(\Omega )} ds + |b|\left( (0, 1) \times \Omega \right) . \end{aligned}$$
(4.8)

Let \(\mathcal {T}^{1}\) be the set of full Lebesgue measure in (0, 1) such that for all \(s\in \mathcal {T}^{1}\)

$$\begin{aligned} \int _{\Omega }|B(s,x)|^{r} dx <\infty . \end{aligned}$$
(4.9)

The set of \(\mathcal {T}^{0}\) which consists of the set of \(s \in (0,1)\) such that

$$\begin{aligned} \int _{\Omega }|B(s,x)|^{r} dx \le \bar{e}^{r}:= \int _{0}^{1} ds \int _{\Omega }|B(s,x)|^{r} dx \end{aligned}$$
(4.10)

is of positive Lebesgue measure.

We use (4.7) to obtain that for any \(H \in C_{c}^{1} (\Omega ;\Lambda ^{k-1}),\) the existence of a set \(\mathcal {T}^{H} \subset \mathcal {T}^{1}\) of full Lebesgue measure in (0, 1) such that

$$\begin{aligned} \int _{\Omega }\langle dH(x); h(s, dx) \rangle = - \int _{\Omega }\langle H(x); B(s, x) \rangle dx \end{aligned}$$
(4.11)

for any \(s \in \mathcal {T}^{H}.\)

Let \(\{F_{n}\}_{n=1}^{\infty }\subset C_{c}^{1}(\Omega ;\Lambda ^{k-1})\) be a dense of \(C_{c}^{1}(\Omega ;\Lambda ^{k-1})\) for the \(||\cdot ||_{C^{1}(\Omega )} \)-norm. Set

$$\begin{aligned} \mathcal {T}:= \cap _{n=1}^{\infty }\mathcal {T}^{F_{n}}. \end{aligned}$$

The set \(\mathcal {T }\cap \mathcal {T}^{0}\) has the same measure as \(\mathcal {T}^{0}\). Let \(t_{0} \in \mathcal {T }\cap \mathcal {T}^{0}\). By Theorems 7.2 and 7.4 [7] (written for \(r \in [2,\infty )\) but extendable to \(r \in (1,2)\)), there is \(\bar{h}_{t_{0}} \in W^{1,r}(\Omega ; \Lambda ^{k})\) such that

$$\begin{aligned} \left\{ \begin{array}{ll} \delta \bar{h}_{t_{0}}= B(t_{0}, \cdot ), \;\; d \bar{h}_{t_{0}} =0 &{}\quad \text {in }\quad \Omega \\ \nu \wedge \bar{h}_{t_{0}}=0 \; &{}\quad \text {on }\partial \Omega . \end{array} \right. \end{aligned}$$

Furthermore, there is a constant C which depends only on \(\Omega \), k and r such that

$$\begin{aligned} \left\| \bar{h}_{t_{0}} \right\| _{W^{1,r}}\le C (\mathcal {L}^{d}(\Omega ))^{\frac{1 }{r}} \Vert B(t_{0}, \cdot ) \Vert _{L^{r}(\Omega )}. \end{aligned}$$
(4.12)

Set

$$\begin{aligned} \bar{h}(s, \cdot ):= h(s, \cdot )-h(t_{0}, \cdot )+ \bar{h}_{t_{0}}. \end{aligned}$$

(i) Observe that (4.11) holds for any \(s \in \mathcal {T}\) and any H which is a point of accumulation on \(\{F_{n}\}.\) Using the fact that \(\{F_{n}\}_{n=1}^{\infty }\) is dense in \(C_{c}^{1}(\Omega ;\Lambda ^{k-1})\) we conclude the proof of (i).

(ii) We exploit Corollary 4.6 and to obtain

$$\begin{aligned} ||\bar{h}(s, \cdot )|| \le ||h(s, \cdot )-h(t_{0}, \cdot )||+ ||\bar{h}_{t_{0}}|| \le |b|\left( (0, 1) \times \Omega \right) + ||\bar{h}_{t_{0}}||. \end{aligned}$$

This, together with (4.12) yields the desired inequality.

(iii) Observe that if \(g \in C_{c}^{1}\left( (0,1) \times \Omega ;\Lambda ^{k}\right) \) then

$$\begin{aligned}&\int _{0}^{1} ds \int _{\Omega }\langle \partial _{s} g(s,x); \bar{h}_{t_{0}}(x) \rangle dx -\int _{0}^{1} ds \int _{\Omega }\langle \partial _{s} g(s,x); h(t_{0}, dx) \rangle \nonumber \\&\quad = \int _{\Omega }\left\langle \bar{h}_{t_{0}}(x); \int _{0}^{1} \partial _{s} g(s,x) ds \right\rangle dx - \int _{\Omega }\left\langle h(t_{0}, dx); \int _{0}^{1} \partial _{s} g(s,x)ds \right\rangle dx=0. \end{aligned}$$
(4.13)

That all is needed to conclude that we may substitute \(\bar{h}\) with h in (4.6). By (i) \(\delta h(t_{0}, \cdot )= B(t_{0}, \cdot )\). Using the definition of \(\bar{h}_{t_{0}}\) we conclude that we may substitute \(\bar{h}\) with h in (4.7). \(\square \)

Definition 4.8

We define \(BV_{*}^{r}(0,1;\Omega )\) to be the set of \(h \in L^{1}\left( (0,1); \mathcal {M}(\Omega ;\Lambda ^{k})\right) \) such that \(\delta h \in L^{r}\left( (0,1); L^{r}(\Omega ; \Lambda ^{k-1})\right) ,\) and there exists \(b\in \mathcal {M}\left( (0,1) \times \Omega ;\Lambda ^{k}\right) \) such that (4.6) holds. We write \(b=\partial _{s} h.\)

Lemma 4.9

Let \((h^{\epsilon })_{\epsilon \in (0,1)} \subset BV_{*}^{r}(0,1;\Omega )\) such that that

$$\begin{aligned} \sup _{\epsilon \in (0,1)} ||\partial _{s} h^{\epsilon }||_{1} + ||\delta h^{\epsilon }||_{r}^{r}<\infty \end{aligned}$$
(4.14)

and

$$\begin{aligned} m_{0}:=\sup _{\epsilon \in (0,1)} \sup _{ s \in (0,1)} ||h^{\epsilon }(s, \cdot )||_{1} <\infty . \end{aligned}$$
(4.15)

Then there exists \(h^{0} \in BV_{*}^{r}(0,1;\Omega )\) such that up to a subsequence the following hold.

  1. (i)

    \((\delta h^{\epsilon })_{\epsilon }\) converges to \(\delta h^{0}\) weakly in \(L^{r}\left( (0,1) \times \Omega ; \Lambda ^{k-1} \right) \).

  2. (ii)

    \((\partial _{s} h^{\epsilon })_{\epsilon }\) converges weak \(*\) to \(\partial _{s} h^{0}\) on \((0,1) \times \Omega \).

  3. (iii)

    Except for countably many \(s \in (0,1),\) \((h^{\epsilon }(s, \cdot ))_{\epsilon }\) converges weak \(*\) to \(h^{0}(s, \cdot )\) on \(\Omega \)

Proof

There are

$$\begin{aligned} b\in \mathcal {M}\left( (0,1) \times \Omega ;\Lambda ^{k}\right) ,\; B \in L^{r}\left( (0,1); L^{r}(\Omega ; \Lambda ^{k-1})\right) ,\;\beta \in \mathcal {M}\left( (0,1) \times \Omega \right) ,\; \beta \ge 0 \end{aligned}$$

and a sequence \(\{ \epsilon _{m}\}_{m}\) decreasing to 0 such that the following hold:

  1. (a)

    \((\delta h^{\epsilon })_{\epsilon }\) converges to B weakly in \(L^{r}\left( (0,1) \times \Omega ; \Lambda ^{k-1} \right) \)

  2. (b)

    \((\partial _{s} h^{\epsilon })_{\epsilon }\) converges weak \(*\) to b on \((0,1) \times \Omega \)

  3. (c)

    \((|\partial _{s} h^{\epsilon }|)_{\epsilon }\) converges weak \(*\) to \(\beta \) on \(\mathbb {R }\times \mathbb {R}^{n}.\)

Write \((0,1) \cap \mathbb {Q}=\{t_{i} \}_{i=1}^{\infty }.\) Since

$$\begin{aligned} ||h^{\epsilon _{m}}(t_{i}, \cdot )|| \le m_{0} \end{aligned}$$

we use a diagonal sequence argument to obtain a subsequence of \((\epsilon _{m})_{m}\), which we continuous to label \((\epsilon _{m})_{m}\), such that for each \(i \in \mathbb {N}\) there exists \(\bar{h}_{i} \in \mathcal {M}(\Omega ;\Lambda ^{k})\) such that \((h^{\epsilon _{m}}(t_{i}, \cdot ))_{m}\) converges weak \(*\) to \(\bar{h}_{i} \) on \(\Omega \).

Let D be the set of \(s \in (0,1)\) such that \(\beta (\{s\} \times \mathbb {R} ^{n})>0.\) Since b is a finite measure, D is at most countable. Let \(s \in (0,1) {\setminus } D\) and let \((t_{i_{j}})_{j}\) be a subsequence of \((t_{i})_{i}\) that converges to s. By Lemma 4.6

$$\begin{aligned} ||h^{\epsilon _{m}}(s, \cdot )-h^{\epsilon _{m}}(t_{i_{j} }, \cdot )|| \le |\partial _{s} h^{\epsilon _{m}}| \left( \left[ \min \{s, t_{i_{j} }\} , \max \{s, t_{i_{j}}\} \right] \times \Omega \right) . \end{aligned}$$
(4.16)

Because \(||h^{\epsilon _{m}}(s, \cdot )|| \le m_{0}\), the set \(\{ h^{\epsilon _{m}}(s, \cdot ) \}_{m}\) admits points of accumulation for the weak \(*\) topology. Let \(h^{0}(s, \cdot )\) be one of these points of accumulation. Letting m tend to \(\infty \) in (4.16) we have

$$\begin{aligned} \left\| h^{0}(s, \cdot )-h_{i_{j}}\right\| \le \beta \left( \left[ \min \{s, t_{i_{j}}\} , \max \{s, t_{i_{j}}\} \right] \times \Omega \right) \end{aligned}$$

and so,

$$\begin{aligned} \limsup _{j \rightarrow \infty } \left\| h^{0}(s, \cdot )-h_{i_{j}}\right\| \le \beta \left( \{s\} \times \bar{\Omega }\right) =0 \end{aligned}$$

Thus, \(\{ h^{\epsilon _{m}}(s, \cdot ) \}_{m}\) admits only one points of accumulation and \((h_{i_{j}})_{j}\) converges weak \(*\) to \(h^{0}(s, \cdot )\). We extend \(s \rightarrow h^{0}(s, \cdot )\) to (0, 1) by setting \(h^{0}(s, \cdot ) \equiv 0\) for \(s \in D\).

Let \(g \in C_{c}^{1}\left( (0,1) \times \Omega ; \Lambda ^{k} \right) .\) Since

$$\begin{aligned} \lim _{m\rightarrow \infty }\int _{\Omega }\langle h^{\epsilon _{m}}(s, x);\partial _{s} g(s,x)\rangle dx =\int _{\Omega }\langle h^{0}(s, dx); \partial _{s} g(s,x) \rangle \end{aligned}$$

and

$$\begin{aligned} \left| \int _{\Omega }\langle h^{\epsilon _{m}}(s, dx); g(s,x) \rangle \right| \le m_{0} ||\partial _{s} g||_{\infty } \end{aligned}$$

for every \(s \in (0,1) {\setminus } D\), we use the dominated convergence theorem to conclude that

$$\begin{aligned} \int _{(0,1) \times \Omega } \langle b(ds, dx); \partial _{s} g(s,x)\rangle&= -\lim _{m\rightarrow \infty }\int _{(0,1) \times \Omega } \langle h^{\epsilon _{m} }(s, x); \partial _{s} g(s,x)\rangle ds dx\\&= -\int _{0}^{1} ds \int _{\Omega }\langle h^{0}(s, dx); \partial _{s} g(s,x)\rangle . \end{aligned}$$

Thus, \(b=\partial h^{0}.\) Similarly, we show that \(\delta h^{0}=B\) and so, modifying \(h^{0}\) on a subset of (0, 1) of null Lebesgue measure \(h^{0} \in BV_{*}^{r}(0,1;\Omega )\). \(\square \)

5 Finsler type metrics

Assume \(\Omega \subset {\mathbb {R}}^n\) is an open bounded convex set, \(p \in (1,\infty )\) and \(rp=r+p.\) Motivated by examples of cost functions c such as the one in Sect. B.1, we relax the condition imposed on the lower bound of \(c^{*}\) in Sect. 3 (cf. 3.2). This allows to extend Theorem 3.8 to cost functions which take on infinite values. Throughout this section,

$$\begin{aligned} c: \Lambda ^{k} \times \Lambda ^{k-1} \rightarrow [0,\infty ] \end{aligned}$$

is lower semicontinuous convex function. We assume that when \(c(\omega , \xi )<\infty \) then

$$\begin{aligned} c(\omega , \xi )=0 \;\;\; \hbox {if and only if} \;\;\; \xi =0 \end{aligned}$$
(5.1)

and for any \(\lambda >0\) we have

$$\begin{aligned} c(\omega , \lambda \xi )=\lambda ^{p} c(\omega , \xi ). \end{aligned}$$
(5.2)

We assume that there are constants \(\gamma _{1}, \gamma _{2}, \gamma _{6}, \gamma _{7}>0\) such that

$$\begin{aligned} c(\omega , \xi ) \ge \gamma _{6}\left( |\omega |^{p} +|\xi |^{p} \right) -\gamma _{7} \end{aligned}$$
(5.3)

and

$$\begin{aligned} \infty > c^{*}(b, B) \ge \gamma _{1}\left( |b|+|B|^{r} \right) -\gamma _{2} \end{aligned}$$
(5.4)

for any \(\omega , b \in \Lambda ^{k}\) and \(\xi , B \in \Lambda ^{k-1}.\) Note that we may have

$$\begin{aligned} \{(\omega , \xi ) \in \Lambda ^{k} \times \Lambda ^{k-1} \; | \; c(\omega , \xi )=\infty \} \not = \emptyset . \end{aligned}$$
(5.5)

Let \(||\cdot ||_{f}\) and \(M_{p}(\cdot , \cdot )\) be defined as in (1.7) and (1.8).

Remark 5.1

Observe the following.

  1. (i)

    In case (5.5) does not hold, then by Lemma B.4 there exists a norm \(\Vert \cdot \Vert _{norm}\) such that \(c(\omega , A) \equiv \Vert A \Vert _{norm}^{p}\) is independent of \(\omega .\) According to [9] the solutions of (1.10) are minimizers of (1.8) and the only minimizers if we further impose that \(\Vert \cdot \Vert _{norm}^{p}\) is strictly convex.

  2. (ii)

    When \(k=n\), which is the case of volume forms, in the current literature, most work studying geodesics of length, deal with either the case when c assumes only finite values (as in Sect. 3) or the case when \(c^{*}(b, B) \in \{0,\infty \}\) for all \((b, B) \in \Lambda ^{k} \times \Lambda ^{k-1}\). It seems obvious that when \(c^{*}(b, B) \in \{0,\infty \}\) (see Remark B.1 for such an example when \(k=2\)), the study of geodesics of optimal length in the set of k-form will only mimic the well-known theory of n-forms. Therefore, in the current manuscript, we keep or focus on the case where (5.4) is satisfied (cf. Sect. B.1 for an example).

For any Borel map \(f: \Omega \rightarrow \Lambda ^{k}\), we define

$$\begin{aligned} c^{\infty }({f}):= \mathrm {ess}\sup _{x, \xi } \left\{ |c\left( f(x), \xi \right) | \; | \; \xi \in \Lambda ^{k-1}),\; |\xi | \le 1, x \in \Omega \right\} \end{aligned}$$

Let

$$\begin{aligned} \bar{f}_{0},\bar{f}_{1}:\Omega \rightarrow \Lambda ^{k} \end{aligned}$$
(5.6)

be Borel maps. When \(1\le k\le n-1,\) we assume that

$$\begin{aligned} \left\{ \begin{array}{ll} c^{\infty }({\bar{f}_{0}}),\;c^{\infty }({\bar{f}_{1}})<\infty &{} \\ d\bar{f}_{0}=d\bar{f}_{1}\equiv 0 &{} \text {in the weak sense in}\;\Omega \\ (\bar{f}_{1}-\bar{f}_{0})\wedge \nu =0 &{} \text {in the weak sense on}\;\partial \Omega . \end{array} \right. \end{aligned}$$
(5.7)

However when \(k=n\), we assume that

$$\begin{aligned} c^{\infty }({\bar{f}_{0}}),\;c^{\infty }({\bar{f}_{1}})<\infty \quad \text {and}\quad \int _{\Omega }(\bar{f}_{0}(x)-\bar{f}_{1} (x))dx=0 \end{aligned}$$
(5.8)

By (5.2) and (5.3)

$$\begin{aligned} \lambda _{6}|f|^{p}\le \left( \lambda _{7}+c^{\infty }({f})\right) , \end{aligned}$$

and so, (5.7) implies that \(|\bar{f}_{0}|,|\bar{f}_{1}|\) are bounded functions. Let \((\bar{f},\bar{A})\) be as in Remark 2.3. The same Remark provides us with a constant \(C_{p,\Omega }\) independent of \(\bar{f}_{0},\bar{f}_{1}\) such that

$$\begin{aligned} ||\bar{A}||_{W^{1,p}}\le C_{p,\Omega }||\bar{f}_{1}-\bar{f}_{0}||_{L^{p} }. \end{aligned}$$
(5.9)

By the convexity property of c,

$$\begin{aligned} c\left( (1-s)\bar{f}_{0}+s\bar{f}_{1},\bar{A}\right) \le (1-s)c(\bar{f}_{0} ,\bar{A})+sc(\bar{f}_{1},\bar{A}) \end{aligned}$$

and so, by the homogeneity with respect to the second variables

$$\begin{aligned} c\left( (1-s)\bar{f}_{0}+s\bar{f}_{1},\bar{A}\right) \le \left( (1-s)c^{\infty }({\bar{f}_{0}})+sc^{\infty }({\bar{f}_{1}})\right) |\bar{A}|^{p} \end{aligned}$$
(5.10)

Recall that \(P^{p}(\bar{f}_{0},\bar{f}_{1})\) is a set of paths connecting \(\bar{f}_{0}\) to \(\bar{f}_{1}\) as given in Definition 2.2. In other words, if \((f, A) \in P^{p}(\bar{f}_{0}, \bar{f}_{1})\) then in the weak sense

$$\begin{aligned} \left\{ \begin{array}{ll} \partial _{s} f +dA \equiv 0 &{} \quad \text {in }\;\; (0,1) \times \Omega \\ A \wedge \nu =0 &{} \quad \text {on }\; (0,1) \times \partial \Omega \\ f(0,\cdot )=\bar{f}_{0}, \; f(1, \cdot )=\bar{f}_{1} &{} \quad \text {on}\; \partial \Omega \end{array} \right. \end{aligned}$$
(5.11)

5.1 A metric on a subset of the set of differential forms

Lemma 5.2

(Reparametrization by arc lengths) Suppose \(c: \Lambda ^{k} \times \Lambda ^{k-1} \rightarrow [0,\infty ]\) is a lower semicontinuous convex function that satisfies (5.1) and (5.2). If \(\bar{f}_{0}\) and \(\bar{f}_{1}\) are such that (5.6) and (5.75.8) hold then

$$\begin{aligned} M_{p}^{p}(\bar{f}_{0}, \bar{f}_{1})=\inf _{(f,A)}\left\{ \mathcal {C }(f, A) \; | \; (f,A) \in P^{p}(\bar{f}_{0}, \bar{f}_{1})\right\} . \end{aligned}$$

Proof

For any \((f,A) \in P^{p}(\bar{f}_{0}, \bar{f}_{1})\) we use Jensen’s inequality to conclude that

$$\begin{aligned} \left( \int _{0}^{1} ds ||A_{s}||_{f_{s}} ds \right) ^{p} \le \mathcal {C}(f, A). \end{aligned}$$

Thus,

$$\begin{aligned} M_{p}^{p}(\bar{f}_{0}, \bar{f}_{1}) \le \inf _{(f,A)}\left\{ \mathcal {C}(f, A) \; | \; (f,A) \in P^{p}(\bar{f}_{0}, \bar{f}_{1})\right\} . \end{aligned}$$

It remains to prove the reverse inequality. Assume without loss of generality that \(M_{p}^{p}(\bar{f}_{0}, \bar{f}_{1})<\infty \) otherwise, there will be nothing to prove. Let \(\epsilon >0\) and let \((f^{\epsilon },A^{\epsilon }) \in P^{p}(\bar{f}_{0}, \bar{f}_{1})\) be such that

$$\begin{aligned} \left( \int _{0}^{1} ds \int _{\Omega }||A_{s}^{\epsilon }||_{f_{s}^{\epsilon }} ds\right) ^{p}< M_{p}^{p}(\bar{f}_{0}, \bar{f}_{1} )+\epsilon . \end{aligned}$$
(5.12)

Define

$$\begin{aligned} L_{\epsilon }:= \int _{0}^{1} (\epsilon + ||A_{s}^{\epsilon }||_{f_{s}^{\epsilon }} )ds , \quad S_{\epsilon }(s):={\frac{1 }{L_{\epsilon }}} \int _{0}^{s} \left( \epsilon + ||A_{l}^{\epsilon }||_{f_{l}^{\epsilon }} \right) dl. \end{aligned}$$

Observe that \(S_{\epsilon }: [0,1] \rightarrow [0,1]\) is a bijection and so has an inverse \(T_{\epsilon }: [0,1] \rightarrow [0,1]\) such that

$$\begin{aligned} \dot{T}_{\epsilon }={\frac{1 }{\dot{S}_{\epsilon }\circ T_{\epsilon }}}= {\frac{L_{\epsilon }}{\epsilon + ||A_{T_{\epsilon }}^{\epsilon }||_{f_{T_{\epsilon }}} }}. \end{aligned}$$
(5.13)

Define

$$\begin{aligned} \tilde{f}(\tau , x)= f^{\epsilon }(T_{\epsilon }(\tau ), x), \quad \tilde{A}(\tau , x)= \dot{T}_{\epsilon }(\tau ) A^{\epsilon }(T_{\epsilon }(\tau ), x). \end{aligned}$$

We have \((\tilde{f}, \tilde{A}) \in P^{p}(\bar{f}_{0}, \bar{f}_{1})\) and

$$\begin{aligned} ||\tilde{A}_{\tau }||_{\tilde{f}_{\tau }}^{p}=\int _{\Omega }c\left( \tilde{f}(\tau , x), \tilde{A}(\tau , x) \right) dx= |\dot{T}_{\epsilon }(\tau )|^{p} \int _{\Omega }c\left( f^{\epsilon }\left( T_{\epsilon }(\tau ), x\right) , A^{\epsilon }\left( T_{\epsilon }(\tau ), x\right) \right) dx. \end{aligned}$$

Thus, using (5.13) we obtain that

$$\begin{aligned} ||\tilde{A}_{\tau }||_{\tilde{f}_{\tau }}^{p}=|\dot{T}_{\epsilon }(\tau )|^{p} \; ||A^{\epsilon }_{T_{\epsilon }(\tau )}||_{f^{\epsilon }_{T_{\epsilon }(\tau )}}^{p}= {\frac{L_{\epsilon }^{p} ||A^{\epsilon }_{T_{\epsilon }(\tau )}||_{f^{\epsilon }_{T_{\epsilon }(\tau )}}^{p} }{(\epsilon + ||A^{\epsilon }_{T_{\epsilon }(\tau )}||_{f^{\epsilon }_{T_{\epsilon }(\tau )}})^{p} }} \le L_{\epsilon }^{p}. \end{aligned}$$

After an integration over (0, 1) we use (5.12) to conclude that

$$\begin{aligned} \inf _{(f,A)}\left\{ \mathcal {C}(f, A) \; | \; (f,A) \in P^{p}(\bar{f}_{0}, \bar{f}_{1})\right\}\le & {} \left( \int _{0}^{1} (\epsilon + ||A^{\epsilon } _{s}||_{f^{\epsilon }_{s}} )ds\right) ^{p}\\\le & {} \left( \left( M_{p}^{p}(\bar{f}_{0}, \bar{f}_{1})+\epsilon \right) ^{\frac{1 }{p}} + \epsilon \right) ^{p} \end{aligned}$$

Letting \(\epsilon \) tend to 0 we have

$$\begin{aligned} \inf _{(f,A)}\left\{ \mathcal {C}(f, A) \; | \; (f,A) \in P^{p}(\bar{f}_{0}, \bar{f}_{1})\right\} \le M_{p}^{p}(\bar{f}_{0}, \bar{f}_{1}). \end{aligned}$$

\(\square \)

Lemma 5.3

Suppose \(c: \Lambda ^{k} \times \Lambda ^{k-1} \rightarrow [0,\infty ]\) is a lower semicontinuous convex function that satisfies (5.1) and (5.2). There exists a constant \(\bar{C}_{\Omega }\) which depends only on \(\Omega \) and s such that if \(\bar{f}_{0}\) and \(\bar{f}_{1}\) are such that (5.6) and (5.75.8) hold then

$$\begin{aligned} M_{p}^{p}(\bar{f}_{0}, \bar{f}_{1}) \le \bar{C}_{\Omega }||\bar{f}_{1} -\bar{f}_{0}||^{p}_{p}. \end{aligned}$$

Proof

Define \((\bar{f}, \bar{A})\) is as in Remark 2.3 (i) and recall that by (ii) of the same Remark, \((\bar{f}, \bar{A}) \in P^{p}(\bar{f}_{0}, \bar{f}_{1})\). We integrate the expressions in (5.10) to obtain

$$\begin{aligned} \mathcal {C}(\bar{f}, \bar{A}) \le {\frac{c^{\infty }({\bar{f}_{0}}) +c^{\infty }({\bar{f}_{1}})}{2}} ||\bar{A}||^{p}_{p} \end{aligned}$$

We first use Lemma 5.2 and then (5.9) to conclude that

$$\begin{aligned} M_{p}^{p}(\bar{f}_{0}, \bar{f}_{1}) \le \mathcal {C}(\bar{f}, \bar{A}) \le \bar{C}_{\Omega }||\bar{f}_{1} -\bar{f}_{0}||^{p}_{p}, \end{aligned}$$

which completes the proof. \(\square \)

Denote by \(\mathcal {H}_{p}\) the set of k-forms \(f \in L^{p}\left( \Omega ;\Lambda ^{k}\right) \) such that

$$\begin{aligned} \left\{ \begin{array}{ll} d f \equiv 0 &{} \quad \text {in the weak sense on }\;\; \bar{\Omega }\\ (f - \bar{f}_{0}) \wedge \nu =0 &{} \quad \text {in the weak sense on }\; \partial \Omega \qquad \qquad \qquad \qquad \qquad \hbox {if} \; 1\le k \le n-1 \end{array} \right. \end{aligned}$$

and

$$\begin{aligned} \left\{ \begin{array}{ll} d f \equiv 0 &{} \text {in the weak sense on }\;\; \bar{\Omega }\\ \int _{\Omega }(f - \bar{f}_{0})dx=0 &{} \qquad \qquad \qquad \qquad \qquad \qquad \qquad \qquad \quad \hbox {if} \; k=n \end{array} \right. \end{aligned}$$

Theorem 5.4

Suppose \(c: \Lambda ^k \times \Lambda ^{k-1} \rightarrow [0,\infty ]\) is a lower semicontinuous convex function that satisfies (5.1), (5.2) and (5.3). Then the following hold.

  1. (i)

    If \(\bar{f}_0\) and \(\bar{f}_1\) satisfy (5.6) and (5.7) then there exists \((f^*, A^*)\) that minimizes \(\mathcal {C}\) and \(\int _0^1 ||A_s||_{f_s}ds\) over \(P^p(\bar{f}_0, \bar{f}_1).\)

  2. (ii)

    The function \(M_p\) in (1.8) is a metric on the set \(\{f \in \mathcal {H}_p \; | \; c^\infty (f)<\infty \}.\)

Proof

(i) follows from Proposition 3.2 and Lemma 5.2.

(ii) Let \(\tilde{f}_{0}, \tilde{f}_{1}, \tilde{f}_{2}\in \mathcal {H}_{p}.\) By (i) and Lemma 5.2 there are \((f^{0}, A^{0}) \in P^{p} (\bar{\omega }_{0}, \bar{\omega }_{1})\) and \((f^{1}, A^{1}) \in P^{p}(\bar{\omega }_{1}, \bar{\omega }_{2})\) such that

$$\begin{aligned} \int _{0}^{1} ||A^{0}_{s}||_{f^{0}_{s}}^{p} ds= \mathcal {C}(f^{0}, A^{0})=M_{p}^{p}(\tilde{f}_{0}, \tilde{f}_{1})= \left( \int _{0}^{1} ||A^{0}_{s}||_{f^{0}_{s}} ds \right) ^{p} \end{aligned}$$
(5.14)

and

$$\begin{aligned} \int _{0}^{1} ||A^{1}_{s}||_{f^{1}_{s}}^{p} ds= \mathcal {C}(f^{1}, A^{1})=M_{p}^{p}(\tilde{f}_{1}, \tilde{f}_{2})= \left( \int _{0}^{1} ||A^{1}_{s}||_{f^{1}_{s}} ds \right) ^{p} \end{aligned}$$
(5.15)

By Lemma 5.3, if \(\tilde{f}_{0}=\tilde{f}_{1}\) then \(M_{p}(\tilde{f}_{0}, \tilde{f}_{1})=0.\) Conversely, \(M_{p}(\tilde{f}_{0}, \tilde{f}_{1})=0\) means

$$\begin{aligned} \int _{0}^{1} ds \int _{\Omega }c(f^{0}_{s}(x), A_{s}^{0}(x)) dx=0, \end{aligned}$$

and so, \(c(f^{0}, A^{0})=0\) almost everywhere on \((0,1) \times \Omega .\) By (5.1) \(A_{0}=0\) almost everywhere on \((0,1) \times \Omega .\) This means \((f^{0},0) \in P^{p}(\tilde{f}_{0}, \tilde{f}_{1})\) and so, \(\tilde{f}_{1}=\tilde{f}_{0}.\)

Setting

$$\begin{aligned} \tilde{f}(s,x)=f^{0}(1-s, x), \quad \tilde{A}(s,x)=-A^{0}(1-s,x), \end{aligned}$$

we have \((\tilde{f}, \tilde{A}) \in P^{p}(\tilde{f}_{1}, \tilde{f}_{0})\) and so,

$$\begin{aligned} M_{p}^{p}(\tilde{f}_{1}, \tilde{f}_{0}) \le \mathcal {C}(\tilde{f}, \tilde{A})= \mathcal {C}(f^{0}, A^{0})=M_{p}^{p}(\tilde{f}_{0}, \tilde{f}_{1}) \end{aligned}$$

By symmetry, the reverse inequality holds and so, \(M_{p}^{p}(\tilde{f}_{1}, \tilde{f}_{0})=M_{p}^{p}(\tilde{f}_{0}, \tilde{f}_{1}).\)

Set

$$\begin{aligned} f(s, x)= \left\{ \begin{array}{ll} f^{0}(2s, x) &{} \quad \text {if }\; 0\le s \le {\frac{1 }{2}} \\ f^{1}(2s-1, x) &{} \quad \text {if }\; {\frac{1 }{2}}\le s \le 1 \end{array} \right. \quad A(s, x)= \left\{ \begin{array}{ll} 2 A^{0}(2s, x) &{}\quad \text {if }\; 0\le s \le {\frac{1 }{2}} \\ 2A^{1}(2s-1, x) &{}\quad \text {if }\; {\frac{1 }{2}}\le s \le 1 \end{array} \right. \end{aligned}$$

We have \((f, A) \in P^{p}(\tilde{f}_{0}, \tilde{f}_{2})\) and

$$\begin{aligned} ||A_{s}||_{f_{s}}= \left\{ \begin{array}{ll} 2 ||A^{0}_{2s}||_{f^{0}_{2s}} &{}\quad \text {if }\; 0\le t \le {\frac{1 }{2}} \\ 2 ||A^{1}_{2s-1}||_{f^{1}_{2s-1}} &{}\quad \text {if }\; {\frac{1 }{2}}\le s \le 1 \end{array} \right. \end{aligned}$$

Hence

$$\begin{aligned} M_{p}(\tilde{f}_{0}, \tilde{f}_{2}) \le \int _{0}^{1} ||A_{s}||_{f_{s}} ds= & {} \int _{0}^{\frac{1 }{2}} 2 ||A^{0}_{2s}||_{f^{0}_{2s}} ds+ \int _{\frac{1 }{2} }^{1} 2 ||A^{1}_{2s-1}||_{f^{1}_{2s-1}} ds\\= & {} M_{p}(\tilde{f}_{0}, \tilde{f}_{1}) + M_{p}(\tilde{f}_{1}, \tilde{f}_{2}) \end{aligned}$$

This concludes the proof of (ii). \(\square \)

5.2 A duality result for non-finite cost function

Remark 5.5

The following hold.

  1. (i)

    By the convexity and lower semicontinuity properties of

    $$\begin{aligned} (b, B) \rightarrow \underline{c}_{\epsilon }(b, B):=c^{*}(b, B) +{\frac{\epsilon }{p}} (|b|^{p}+|B|^{p}), \end{aligned}$$

    setting \(c_{\epsilon }:=(\underline{c}_{\epsilon })^{*}\), we have \(c_{\epsilon }^{*}=\underline{c}_{\epsilon }.\)

  2. (ii)

    Observe that since \(c^{*}\) is convex, \(c^{*}_{\epsilon }\) is strictly convex. Furthermore,

    $$\begin{aligned} c^{*}_{\epsilon }\ge c^{*}\quad \hbox {and} \quad c_{\epsilon }\le c. \end{aligned}$$
  3. (iii)

    By (5.4) there is a constant \(\gamma _{3}^{\epsilon }>0\) depending on \(\epsilon >0\) such that

    $$\begin{aligned} -\gamma _{2} + {\frac{\epsilon }{p}} (|b|^{p}+|B|^{p}) \le c^{*}_{\epsilon }(b, B) \le \gamma _{7} + \gamma _{3}^{\epsilon }\left( |b|+|B|^{r} \right) \end{aligned}$$
  4. (iv)

    By (5.3) there are constants \(\gamma _{6}^{*}>0\) and \(\gamma _{7}^{*} \ge 0\) independent of \(\epsilon \in (0,1)\) such that

    $$\begin{aligned} c_{\epsilon }(\omega , \xi ) \ge \gamma _{6}^{*} \left( |\omega |^{p}+|\xi |^{p} \right) -\gamma _{7}^{*}. \end{aligned}$$
  5. (v)

    Using the notation of Sect. 3, since \(c^{*}\) satisfies (iii), Proposition 3.7 asserts the existence of \(h_{\epsilon }\) that maximizes \(\mathcal {D}_{\epsilon }\) over \(\mathbf {B}^{r}\left( (0,1) \times \Omega ; \Lambda ^{k} \right) \). By Theorem 3.8 there exists \((f^{\epsilon }, A^{\epsilon })\) which minimizes \(\mathcal {C}_{\epsilon }\) over \(P^{p}(\bar{f}_{0}, \bar{f}_{1})\). Furthermore, \(\mathcal {D}_{\epsilon }(h_{\epsilon })=\mathcal {C}_{\epsilon }(f^{\epsilon }, A^{\epsilon })\). Since \(c^{*}_{\epsilon }\) is strictly convex, \(c_{\epsilon }\) is continuously differentiable and so, Theorem 3.8 gives

    $$\begin{aligned} (f^{\epsilon }, A^{\epsilon }) \in \partial c^{*}_{\epsilon }(\partial _{s} h_{\epsilon }, \delta h_{\epsilon }) \quad \hbox {i.e.} \quad (\partial _{s} h_{\epsilon }, \delta h_{\epsilon })= \nabla c_{\epsilon }(f^{\epsilon }, A^{\epsilon }). \end{aligned}$$

Theorem 5.6

Assume c satisfies (5.1), (5.2) and (5.3) and \(c^*\) satisfies (5.4). We assume that \(\bar{f}_0, \bar{f}_1 \in C_0(\Omega , \Lambda ^k)\) are such that (5.6), (5.7) hold and there exists \(\epsilon _0>0\) such that \(|\bar{f}_0|, |\bar{f}_1| \le \gamma _1-\epsilon _0\).Then

$$\begin{aligned} \max _{h \in BV_{*}^r(0,1;\Omega )} \mathcal {D}(h)=\min _{(f, A) \in P^p(\bar{f}_0, \bar{f}_1)} \mathcal {C}(f, A). \end{aligned}$$
(5.16)

Proof

1. Let \((f^{\epsilon }, A^{\epsilon })\) and \(h^{\epsilon }\) be the optima in Remark 5.5. We first use the minimality property of \((f^{\epsilon }, A^{\epsilon })\) and then use Remark 5.5 (ii) to conclude that

$$\begin{aligned} \mathcal {C}_{\epsilon }(f^{\epsilon }, A^{\epsilon }) \le \mathcal {C}_{\epsilon }(\bar{f}, \bar{A}) \le \mathcal {C}(\bar{f}, \bar{A})<\infty . \end{aligned}$$

This, together with Remark 5.5 (iv) implies

$$\begin{aligned} \sup _{\epsilon \in (0,1)} ||f^{\epsilon }||_{p}+ ||A^{\epsilon }||_{p} <\infty . \end{aligned}$$

Thus, up to a subsequence \((f^{\epsilon })_{\epsilon }\) converges weakly in \(L^{p}((0,1) \times \Omega ; \Lambda ^{k})\) to some \(f^{0}\) and \((A^{\epsilon })_{\epsilon }\) converges weakly in \(L^{p}((0,1) \times \Omega ; \Lambda ^{k-1})\) to some \(A^{0}.\) For any \(b \in C_{0}((0,1) \times \Omega ; \Lambda ^{k})\) and \(B \in C_{0}((0,1) \times \Omega ; \Lambda ^{k-1})\) we have

$$\begin{aligned} \liminf _{\epsilon \rightarrow 0^{+}} \mathcal {C}_{\epsilon }(f^{\epsilon }, A^{\epsilon })&\ge \liminf _{\epsilon \rightarrow 0^{+}} \int _{(0,1) \times \Omega } \left( \langle f^{\epsilon }; b \rangle + \langle A^{\epsilon }; B \rangle -c^{*}_{\epsilon }(b, B)\right) ds dx\\&= \int _{(0,1) \times \Omega } \left( \langle f^{0}; b \rangle + \langle A^{0}; B \rangle -c^{*}(b, B)\right) ds dx. \end{aligned}$$

Thus, since \(c^{*}\) takes on only finite values, maximizing over (bB), we can use Proposition C.5 (iii) to conclude that

$$\begin{aligned} \liminf _{\epsilon \rightarrow 0^{+}} \mathcal {C} _{\epsilon }(f^{\epsilon }, A^{\epsilon }) \ge \mathcal {C}(f^{0}, A^{0}). \end{aligned}$$
(5.17)

Recall the expression of \({\mathcal {D}}_{\epsilon }\) in (3.9), use the maximality property of \(h^{\epsilon }\) and (5.4) to obtain that

$$\begin{aligned} \gamma _{2} \mathcal {L}^{n}(\Omega )\ge -\mathcal {D}_{\epsilon }(0) \ge -\mathcal {D}_{\epsilon }(h^{\epsilon }) \ge \epsilon _{0} ||\partial _{s} h^{\epsilon }||_{1} + \gamma _{1} ||\delta h^{\epsilon }||_{r}^{r} - ||\bar{A}||_{p} \; ||\delta h^{\epsilon }||_{r} -\gamma _{2} \mathcal {L}^{d}(\Omega ). \end{aligned}$$

Thus, (4.14) holds. Thanks to Lemma 4.7, we may assume without loss of generality that (4.14) holds. We use Lemma 4.9 to conclude that there exists \(h^{0} \in BV_{*}^{r}(0,1;\Omega )\) such that up to a subsequence

  1. (i)

    \((\delta h^{\epsilon _{m}})_{m}\) converges to \(\delta h^{0}\) weakly in \(L^{r}\left( (0,1) \times \Omega ; \Lambda ^{k-1} \right) \).

  2. (ii)

    \((\partial _{s} h^{\epsilon _{m}})_{m}\) converges weak \(*\) to \(\partial _{s} h^{0}\) on \((0,1) \times \Omega )\).

  3. (iii)

    For \(\mathcal {L}^{1}\)-almost every \(s \in (0,1),\) \((h^{\epsilon _{m}}(s, \cdot ))_{m}\) converges weak \(*\) to \(h^{0}(s, \cdot )\) on \(\Omega \)

Since \(c^{*}_{\epsilon }\ge c^{*}\),

$$\begin{aligned} \liminf _{\epsilon \rightarrow 0^{+}} \int _{(0,1) \times \Omega } c_{\epsilon }^{*}(\partial _{s} h^{\epsilon }, \delta h^{\epsilon }) ds dx \ge \liminf _{\epsilon \rightarrow 0^{+}} \int _{(0,1) \times \Omega } c^{*}(\partial _{s} h^{\epsilon }, \delta h^{\epsilon }) dx. \end{aligned}$$
(5.18)

By Theorem 3.3.1 [5] and the convergence in (i) and (ii), we have

$$\begin{aligned} \liminf _{\epsilon \rightarrow 0^{+}} \int _{(0,1) \times \Omega } c^{*}(\partial _{s} h^{\epsilon }, \delta h^{\epsilon }) ds dx \ge \int _{(0,1) \times \Omega } c^{*}(\partial _{s} h^{0}, \delta h^{0}) \end{aligned}$$
(5.19)

The integral of \(c^{*}(\partial _{s} h^{0}, \delta h^{0})\) needs to be interpreted as in Definition 4.2 which involves the recession function \(\bar{c}^{*}.\) Combining (5.18) and (5.19) we obtain

$$\begin{aligned} \liminf _{\epsilon \rightarrow 0^{+}} \int _{(0,1) \times \Omega } c_{\epsilon }^{*}(\partial _{s} h^{\epsilon }, \delta h^{\epsilon }) ds dx \ge \int _{(0,1) \times \Omega } c^{*}(\partial _{s} h^{0}, \delta h^{0}) . \end{aligned}$$
(5.20)

Recall that we can assume without loss of generality that \(s \rightarrow h^{0}(s, \cdot )\) is left continuous at 1 and right continuous at 0. We use the trace operator in Sect. 4.2, and combine (4.14) with (4.15) to obtain that

$$\begin{aligned}&\lim _{m \rightarrow \infty }\int _{\Omega } \langle \bar{f}_{1}(x) ; h^{\epsilon _{m}}(1, x) \rangle dx - \int _{\Omega } \langle \bar{f}_{0}(x) ; h^{\epsilon _{m}}(0, x) \rangle dx\nonumber \\&\quad = \int _{ \Omega } \langle \bar{f}_{1}(x) ; h^{0}(1, dx) \rangle - \int _{ \Omega } \langle \bar{f}_{0}(x) ; h^{0}(0, dx) \rangle . \end{aligned}$$
(5.21)

Rearranging the expressions in the identify \(\mathcal {C}_{\epsilon }(f^{\epsilon }, A^{\epsilon })=\mathcal {D}_{\epsilon }(h^{\epsilon })\) we have

$$\begin{aligned}&\int _{(0,1) \times \Omega } \left( c_{\epsilon }(f^{\epsilon }, A^{\epsilon }) + c_{\epsilon }^{*}(\partial _{s} h^{\epsilon }, \delta h^{\epsilon }) \right) ds dx\nonumber \\&\quad = \int _{\Omega } \langle \bar{f}_{1}(x) ; h^{\epsilon }(1, dx) \rangle - \int _{\Omega } \langle \bar{f}_{0}(x) ; h^{\epsilon }(0, dx) \rangle \end{aligned}$$
(5.22)

Thus, using (5.17), (5.20) and (5.21), together with the fact that

$$\begin{aligned}&\liminf _{\epsilon \rightarrow 0^{+}} \int _{0}^{1} ds \int _{\Omega }\left( c_{\epsilon }(f^{\epsilon }, A^{\epsilon }) dx+ \liminf _{\epsilon \rightarrow 0^{+}} \int _{0}^{1} ds \int _{\Omega }c_{\epsilon }^{*} (\partial _{s} h^{\epsilon }, \delta h^{\epsilon }) \right) dx\\&\quad \le \liminf _{\epsilon \rightarrow 0^{+}} \left( \int _{0}^{1} ds \int _{\Omega }\left( c_{\epsilon }(f^{\epsilon }, A^{\epsilon }) dx+ \int _{0}^{1} ds \int _{\Omega }c_{\epsilon }^{*}(\partial _{s} h^{\epsilon }, \delta h^{\epsilon }) \right) dx \right) , \end{aligned}$$

we obtain

$$\begin{aligned} \mathcal {C}(f^{0}, A^{0})+ \int _{(0,1) \times \bar{\Omega }} c^{*}(\partial _{s} h^{0}, \delta h^{0}) \le \int _{ \Omega } \langle \bar{f}_{1}(x) ; h^{0}(1, dx) \rangle - \int _{ \Omega } \langle \bar{f}_{0}(x) ; h^{0}(0, dx) \rangle . \end{aligned}$$
(5.23)

This means

$$\begin{aligned} \mathcal {C}(f^{0}, A^{0}) \le \mathcal {D}(h^{0}). \end{aligned}$$
(5.24)

2. We claim that \(\mathcal {C}(f, A) \ge \mathcal {D}(h)\) for any \((f, A) \in P^{p}(\bar{f}_{0}, \bar{f}_{1})\) and any \(h \in BV_{*}^{r}(0,1;\Omega )\). Observe that (5.3) and (5.4) imply that \(C:=c^{*}\) satisfies (C.1) and (C.2). By the assumption on c, we have \(C^{*}\ge C^{*}(0)=0.\) Let \(h^{\epsilon }_{l} \in C^{\infty }(\Omega ; \Lambda ^{k})\) and \(h_{l}\in BV_{*}^{r}(0,1;\Omega _{l})\) be the approximations of h as defined by (D.2) in Section D. Here, \(\Omega _{l}\) is the l-neighborhood of \(\Omega .\) We have

$$\begin{aligned} \mathcal {C}(f, A)&\ge \int _{O} \left( \langle f; \partial _{s} h^{\epsilon }_{l} \rangle + \langle A; \delta h^{\epsilon }_{l} \rangle dx\right) -\int _{O} c^{*}(\partial _{s} h^{\epsilon }_{l}, \delta h^{\epsilon }_{l}) dsdx\nonumber \\&=\int _{\Omega }\left( \langle f_{1}(x); h^{\epsilon }_{l}(1, x)\rangle -\langle f_{0}(x); h^{\epsilon }_{l}(0, x) \rangle \right) dx-\int _{O} c^{*} (\partial _{s} h^{\epsilon }_{l}, \delta h^{\epsilon }_{l}) dsdx. \end{aligned}$$

Letting \(\epsilon \) tend to 0 in Lemmas D.3 and D.4 we obtain

$$\begin{aligned} \mathcal {C}(f, A) \ge \int _{\Omega }\left( \langle f_{1}(x); h_{l}(1, x)\rangle -\langle f_{0}(x); h_{l}(0, x) \rangle \right) dx-\int _{O_{l}} c^{*}(\partial _{s} h_{l}, \delta h_{l}) . \end{aligned}$$

Letting l tend to 0 in Lemmas D.3 and D.4 we obtain \(\mathcal {C}(f, A) \ge \mathcal {D}(h).\) This, together with (5.24) concludes the proof of the Theorem. \(\square \)